You are on page 1of 133

Assignment Set- 1

Master of Business Administration-MBA Semester 1

MB0038 – Management Process and Organization Behavior

Q.1 Write a note on the managerial roles and skills?

ANS: According to Mintzberg (1973), managerial roles are as follows:


1. Informational roles
2. Decisional roles
3. Interpersonal roles

1. Informational roles: This involves the role of assimilating and disseminating


information as and when required. Following are the main sub-roles, which
managers often perform:
a. Monitor – collecting information from organizations, both from inside and
outside of the organization
b. Disseminator – communicating information to organizational members
c. Spokesperson – representing the organization to outsiders
2. Decisional roles: It involves decision making. Again, this role can be sub-
divided in to the following:
a. Entrepreneur – initiating new ideas to improve organizational performance
b. Disturbance handlers – taking corrective action to cope with adverse situation
c. Resource allocators – allocating human, physical, and monetary resources
d. Negotiator – negotiating with trade unions, or any other stakeholders
3. Inter`personal roles: This role involves activities with people working in the
organization. This is supportive role for informational and decisional roles.
Interpersonal roles can be categorized under three sub-headings:
a. Figurehead – Ceremonial and symbolic role
b. Leadership – leading organization in terms of recruiting, motivating etc.
c. Liaison – liasoning with external bodies and public relations activities.

Management Skills: Katz (1974) has identified three essential management skills:
technical, human, and conceptual.

Technical skills: The ability is to apply specialized knowledge or expertise. All


jobs require some specialized expertise, and many people develop their technical
skills on the job. Vocational and on-the-job training programs can be used to
develop this type of skill.
Human Skill: This is the ability to work with, understand and motivate other
people (both individually and a group). This requires sensitivity towards others
issues and concerns. People, who are proficient in technical skill, but not with
interpersonal skills, may face difficulty to manage their subordinates. To acquire
the Human Skill, it is pertinent to recognize the feelings and sentiments of others,
ability to motivate others even in adverse situation, and communicate own feelings
to others in a positive and inspiring way.
Conceptual Skill: This is an ability to critically analyze, diagnose a situation and
forward a feasible solution. It requires creative thinking, generating options and
choosing the best available option.
Q.2 Explain the social learning theory in details?

ANS: One of the most influential learning theories, the Social Learning Theory
(SLT), was formulated by Albert Bandura. It encompasses concepts of traditional
learning theory and the operant conditioning of B.F. Skinner.

However, the theory strongly implies that there are types of learning wherein direct
reinforcement is not the causal mechanism; rather, the so called social element can
result to the development of new learning among individuals. Social Learning
Theory has been useful in explaining how people can learn new things and develop
new behaviors by observing other people. It is to assume, therefore, that Social
Learning Theory is concerned on observational learning process among people.

A. Basic Concepts

1. Observational LearningThe Social Learning Theory says that people can


learn by watching other people perform the behavior. Observational learning
explains the nature of children to learn behaviors by watching the behavior
of the people around them, and eventually, imitating them. With the ―Bobo
Doll‖ experiment(s), Bandura included an adult who is tasked to act
aggressively toward a Bobo Doll while the children observe him. Later,
Bandura let the children play inside a room with the Bobo Doll. He affirmed
that these children imitated the aggressive behavior toward the doll, which
they had observed earlier.

After his studies, Bandura was able to determine 3 basic models of


observational learning, which include:
a. A Live Model, which includes an actual person performing a
behavior.
b. A Verbal Instruction Model, which involves telling of details and
descriptions of a behavior.
c. A Symbolic Model, which includes either a real or fictional character
demonstrating the behavior via movies, books, television, radio,
online media and other media sources.
2. The state of mind (mental states) is crucial to learning.In this concept,
Bandura stated that not only external reinforcement or factors can affect
learning and behavior. There is also what he called intrinsic reinforcement,
which is in a form of internal reward or a better feeling after performing the
behavior (e.g. sense of accomplishment, confidence, satisfaction, etc.)
3. Learning does not mean that there will be a change in the behavior of an
individual.

B. Modeling Process

The Modeling Process developed by Bandura helps us understand that not all
observed behaviors could be learned effectively, nor learning can necessarily result
to behavioral changes. The modeling process includes the following steps in order
for us to determine whether social learning is successful or not:
Step 1: Attention

Social Cognitive Theory implies that you must pay attention for you to learn. If
you want to learn from the behavior of the model (the person that demonstrates the
behavior), then you should eliminate anything that catches your attention other
than him. Also, the more interesting the model is, the more likely you are to pay
full attention to him and learn.

Step 2: Retention

Retention of the newly learned behavior is necessary. Without it, learning of the
behavior would not be established, and you might need to get back to observing the
model again since you were not able to store information about the behavior.

Step 3: Reproduction

When you are successful in paying attention and retaining relevant information,
this step requires you to demonstrate the behavior. In this phase, practice of the
behavior by repeatedly doing it is important for improvement.

Step 4: Motivation

Feeling motivated to repeat the behavior is what you need in order to keep on
performing it. This is where reinforcement and punishment come in. You can be
rewarded by demonstrating the behavior properly, and punished by displaying it
inappropriately
Q.3 Explain the Big 5 model of personality?

Ans : The big five model of personality is designed to bring out behaviors an
individual expresses in his dealings with people as well as in their response to
changes in circumstances as well as the environment. Generally, these five factors
of personality are used to come up with a description of the human personality. In
this text, I explain Tisha‘s findings as well as look at how the predictions could be
used to come up with a prediction of her success as a manager.

The big five model of personality

 Openness to experience

This factor acts to distinguish conventional individuals from those who are in one
way or the other imaginative. Traits linked to individuals who are open to
experience include intellectual curiosity, sensitivity to beauty as well as
responsiveness to art. According to Cattell, H.E.P et al. (2007), such people are
very creative and they are very likely to be more accommodating to beliefs
considered unconventional. Such people are hence more likely to be
accommodating to the views of others in the organizational setting and they are
more likely to encourage innovation at the workplace. Tisha‘s high score shows
that she will be more straightforward and more likely see the simple aspects of
complex situations.


 Conscientiousness

This factor shows an individuals ability and drive to achieve goals and strive for
achievement. It also indicates that an individual has self discipline and prefers
planned behavior as opposed to spontaneous behavior. Grucza et al. (2007) notes
that individuals who score highly in conscientiousness have a good impulse control
and tend to be goal oriented. Tisha‘s very high performance in Conscientiousness
shows that she is organized and very attentive to detail. In an organizational
setting, Tisha would be more likely to strive to ensure that organizational goals and
objectives are met.

 Extraversion

This factor consists of emotions that can be said to be positive as well as an


industrious personality and ability to relate well with others. Cattell, H.E.P et al.
(2007) notes that extraverts enjoy other peoples company and tend to be very
active in group efforts. Tisha‘s high score in extraversion indicates that in the work
setting as a manager, she would tend to encourage team work. She will also
dominate board and other meetings and have a high chance of pushing her
proposals and suggestions forward.

 Agreeableness

This factor is characterized by compassion and cooperation towards others as


opposed to antagonism tendencies. Cattell, H.E.P et al. (2007) notes that such
individuals are easy to get along with and are more likely to be very
accommodating to the views of others.

Tisha‘s moderately high score on this factor shows that though she will be
accommodate others view, she will expect her views also to be taken into
consideration. She is likely to trust her subordinates more.

 Neuroticism

This factor involves a high likelihood to get angry and have other negative
emotions like anxiety. In most cases, this characteristic is linked to emotional
instability. Individuals scoring low on neuroticism will experience bouts of mood
swings triggered by frustrations by minor issues at the workplace (Grucza et al.
2007). Tisha‘s low score on this factor shows she will be more likely to cope with
problems as a manager and she will tend to be calmer while handling difficult
situations.

Q.4 What are the different factors influencing perception?

Ans : Perception is our sensory experience of the world around us and involves
both the recognition of environmental stimuli and action in response to these
stimuli. Through the perceptual process, we gain information about properties and
elements of the environment that are critical to our survival.

A number of factors operate to shape and sometimes distort perception These


factors can reside:
i) In the perceiver
ii) In the Object or target being perceived or
iii) In the context of the situation in which the perception is made.
1. Characteristics of the Perceiver: Several characteristics of the perceiver can
affect perception. When an individual looks at a target and attempts to interpret
what he or she stands for, that interpretation is heavily influenced by personal
characteristics of the individual perceiver. The major characteristics of the
perceiver influencing perception are:
a) Attitudes: The perciver's attitudes affect perception. For example, Mr. X is
interviewing candidates for a very important position in his organization - a
position that requires negotiating contracts with suppliers, most of whom are male.
Mr. X may feel that women are not capable of holding their own in tough
negotiations. This attitude with doubtless affect his perceptions of the female
candidates he interviews.
b) Moods: Moods can have a strong influence on the way we perceive someone.
We think differently when we are happy than we do when we are depressed. In
addition, we remember information that is consistent with our mood state better
than information that is inconsistent with our mood state. When in a positive mood,
we form more positive impressions of other. When in a negative mood, we tend to
evaluate others unfavourably.
c) Motives: Unsatisfied needs or motives stimulate individuals and may exert a
strong influence on their perceptions. For example, in an organizational context, a
boss who is insecure perceives a sub ordinate's efforts to do an outstanding job as a
threat to his or her own position. Personal insecurity can be translated into the
perception that others are out to "get my job", regardless of the intention of the
subordinates.
d) Self - Concept: Another factor that can affect social perception is the perceivers
self-concept. An individual with a positive self-concept tends to notice positive
attributes in another person. In contrast, a negative self-concept can lead a
perceiver to pick out negative traits in another person. Greater understanding of
self allows us to have more accurate perceptions of others.

e) Interest: The focus of our attention appears to be influenced by our interests.


Because our individual interests differ considerably, what one person notices in a
situation can differ from what other perceive. For example, the supervisor who has
just been reprimanded by his boss for coming late is more likely to notice his
colleagues coming late tomorrow than he did last week.
f) Cognitive structure: Cognitive structure, an individual's pattern of thinking, also
affects perception. Some people have a tendency to perceive physical traits, such
as height, weight, and appearance, more readily. Cognitive complexity allows a
person to perceive multiple characteristics of another person rather than attending
to just a few traits.
g) Expectations: Finally, expectations can distort your perceptions in that you will
see what you expect to see. The research findings of the study conducted by
Sheldon S Zalking and Timothy W Costello on some specific characteristics of the
perceiver reveal
i) Knowing oneself makes it easier to see others accurately.
ii) One's own characteristics affect the characteristics one is likely to see in other.
iii) People who accept themselves are more likely to be able to see favourable
aspects of other people.
iv) Accuracy in perceiving others is not a single skill.
These four characteristics greatly influence how a person perceives other int he
environmental situation.

2) Characteristics of the Target : Characteristics in the target that is being observed


can affect what is perceived. Physical appearance pals a big role in our perception
of others. Extremely attractive or unattractive individuals are more likely to be
noticed in a group than ordinary looking individuals. Motions, sound, size and
other attributes of a target shape the way we see it.
Verbal Communication from targets also affects our perception of them. Nonverbal
communication conveys a great deal of information about the target. The perceiver
deciphers eye contact, facial expressions, body movements, and posture all in a
attempt to form an impression of the target.

3) Characteristics of the Situation: The situation in which the interaction between


the perceiver and the target takes place, has an influence on the perceiver's
impression of the target. The strength of the situational cues also affects social
perception. Some situations provide strong cues as to appropriate behaviour. In this
situation, we assume that + i.e individual's behaviours can be accounted for by the
situation, and that it may not reflect the individual's disposition.

Q.5 Write a note on contemporary work cohort?

ANS: Contemporary Work Cohort

Contemporary Work Cohort, proposed by Robbins (2003) divides the work force
into different groups depending on the era or period in which they have entered
into work. It stresses upon individuals‘ values which reflect the societal values of
the period in which they grew up.

The cohorts and the respective values have been listed below:

1.Veterans—Workers who entered the workforce from the early 1940s through the
early 1960s and exhibited the following value orientations:
1. They were influenced by the Great Depression and World War II
2. Believed in hard work
3. Tended to be loyal to their employer
4. Terminal values: Comfortable life and family security

2.Boomers—Employees who entered the workforce during the 1960s through the
mid1980s belonged to this category and their value orientations were:

a. Influenced heavily by John F. Kennedy, the civil rights and feminist movements,
the Beatles, the Vietnam War, and baby boom competition.
b. Distrusted authority, but gave a high emphasis on achievement and material
success.
c. Organizations who employed them were vehicles for their careers.
d. Terminal values: sense of accomplishment and social recognition.

3.Xers—began to enter the workforce from the mid1980s.


They cherished the following values:
a. Shaped by globalization, two career parents, MTV, AIDS, and computers.
b. Value flexibility, life options, and achievement of job satisfaction.
c. Family and relationships were important and enjoyed team oriented work.
d. Less willing to make personal sacrifices for employers than previous
generations.
e. Terminal values: true friendship, happiness, and pleasure

4.Ne xte rs—most recent entrants into the workforce.


a. Grew up in prosperous times, have high expectation, believe in themselves, and
confident in their ability to succeed.
b. Never ending search for ideal job; see nothing wrong with job hopping.
c. Seek financial success.
d. Enjoy team work, but are highly self reliant.
e. Terminal values: freedom and comfortable life.

Q.6 What are the special issues in motivation? Discuss


Ans : Some of the special issues in motivation are discussed below.

Motivating Professionals
The professional employees likely to seek more intrinsic satisfaction from their
work than blue-collar employees. They generally have strong and long term
commitments to their field of expertise are perhaps more loyal to their profession
than to their employer. They need to regularly update their knowledge, and their
commitment to their profession. Therefore, extrinsic factors such as money and
promotions would be low on their priority list. Rather, job challenge tends to be
ranked high. They like to tackle problems and find solutions.
Managerial Implications:
 Provide them with ongoing challenging projects.
 Give them autonomy to follow their interests and allow them to structure
their work.
 Reward them with educational opportunities.
 Also reward them with recognition.

Motivating temporary Workers:


Temporary workers may be motivated if:
 They are provided with permanent job opportunity
 The opportunity for training is provided to them
 Provide equitable pay.

Motivating Low Skilled Service Workers:


One of the most challenging managerial tasks in to motivate low skilled workers
who are involved in repetitive physical work, where higher education and skills are
not required. For this category of people, flexible work schedules and higher pay
package may be proved effective motivational factors. Motivating Low-Skilled
Service Workers involves:-
 Recruit widely.
 Increase pay and benefits.
 Make jobs more appealing.
Motivating People Doing Highly Repetitive Tasks:
 Recruit and select employees that fit the job.
 Create a pleasant work environment.
 Mechanize the most distasteful aspects of the job.
Assignment Set- 2

Q.1 Explain the theories of emotion?

ANS: Psychologists have proposed a number of theories about the origins and
function of emotions. The theorists behind the dissenting views do agree on one
thing, however: emotion has a biological basis. This is evidenced by the fact that
the amygdala (part of the limbic system of the brain), which plays a large role in
emotion, is activated before any direct involvement of the cerebral cortex (where
memory, awareness, and conscious "thinking" take place).

In the history of emotion theory, four major explanations for the complex mental
and physical experiences that we call "feelings" have been put forward. They are:
the James-Lange theory in the 1920's, the Cannon-Bard theory in the 1930's, the
Schacter-Singer theory in the 1960's, and most recently the Lazarus theory,
developed in the 1980's and ‗90's.

The James-Lange Theory

The James-Lange theory proposes that an event or stimulus causes a physiological


arousal without any interpretation or conscious thought, and you experience the
resulting emotion only after you interpret the physical response.

Example:

You're late leaving work, and as you head across the parking lot to your car, you
hear footsteps behind you in the dark. Your heart pounds and your hands start to
shake. You interpret these physical responses as fear.
The Cannon-Bard Theory

The Cannon-Bard theory, on the other hand, suggests that the given stimulus
evokes both a physiological and an emotional response simultaneously, and that
neither one causes the other.

Example:

You're home alone and hear creaking in the hallway outside your room. You begin
to tremble and sweat, and you feel afraid.

The Schacter-Singer Theory

The Schachter-Singer theory takes a more cognitive approach to the issue. Schacter
and Singer believe that an event causes physiological arousal, but that you must
then identify a reason for the arousal before you label the emotion.

Example:

You're taking the last bus of the night, and you're the only passenger. A single man
gets on and sits in the row behind you. When your stop comes around, he also gets
off the bus. He's walking behind you. You feel tingles down your spine with a rush
of adrenaline. You know that there have been several muggings in your city over
the past few weeks, so you feel afraid.
The Lazarus Theory

The Lazarus theory builds on the Schacter-Singer theory, taking it to another level.
It proposes that when an event occurs, a cognitive appraisal is made (either
consciously or subconsciously), and based on the result of that appraisal, an
emotion and physiological response follow.

Example:

You're buying a few last-minute items at the gas station, when two young men in
hooded sweatshirts enter the store in a hurry, with their hands in their jacket
pockets. You think perhaps they're here to rob the place, so you get scared, and
your feel like you might throw up.

While each of these theories is based in research, there is no absolute proof as yet
how emotions arise in our bodies and minds, or what determines our own
individual experiences of them. What we do know is that feelings are a powerful
force to be reckoned with, and should never be belittled.

Q.2 Discuss the techniques of decision making in groups?

ANS : Planning for Decision Making

While decision making without planning is fairly common, it is often not pretty.
The terms used to describe it--crisis management, putting out fires, seat-of-the-
pants governing--all reveal the inelegance and awkwardness of this way of life.
Planning allows decisions to be made in a much more comfortable and intelligent
way. Planning even makes decisions easier by providing guidelines and goals for
the decision. We might even say that planning is a type of decision simplification
technique (see the discussion of these techniques below).

Decision makers will find four major benefits to planning:

1. Planning allows the establishment of independent goals. The vision which


will shape the decisions is set apart from surrounding events. Decisions are not
made only as reactions to external stimuli. "Management by firefighting" is
replaced by a conscious and directed series of choices. Managers now steer the
organization, individuals now steer their lives, rather than being steered by external
forces. Sometimes the difference between planning and not planning is described
as "proactive" (taking control of the situation) versus "reactive" (responding to
stimuli).

2. Planning provides a standard of measurement. A plan provides something to


measure against, so that you can discover whether or not you are achieving or
heading toward your goals. As the proverb says, If you don't know where you're
going, it doesn't matter which way you go.

3. Planning converts values to action. When you are faced with a decision, you
can consult your plan and determine which decision will help advance your plan
best. Decisions made under the guidance of planning can work together in a
coherent way to advance company or individual goals.

Planning is useful in emergency situations, too. When a crisis arises, a little


thought about the overall plan will help determine which decision to make that will
not only help resolve the crisis but will also help advance the overall plan. Without
a plan, crises are dealt with haphazardly and decisions are made which may
ultimately be in conflict with each other.
4. Planning allows limited resources to be committed in an orderly way.
Budgets, time, effort, manpower--all are limited. Their best use can be made when
a plan governs their use.

A simple example would be planning to buy a house or a car. Rather than having
to decide between buying the item right now with all cash or never having it, you
can plan to buy it over several years by making payments. Or, you might combine
this plan with the plan to buy a smaller house and add rooms later as they could be
afforded. By planning you can thus accomplish things that might otherwise look
impossible.

Decision Levels

We all recognize that some decisions are more important than others, whether in
their immediate impact or long term significance. As a means of understanding the
significance of a decision so that we can know how much time and resources to
spend on it, three levels of decision have been identified:

1. Strategic. Strategic decisions are the highest level. Here a decision concerns
general direction, long term goals, philosophies and values. These decisions are the
least structured and most imaginative; they are the most risky and of the most
uncertain outcome, partly because they reach so far into the future and partly
because they are of such importance.

For example: Decisions about what to do with your life, what to learn, or what
methods to use to gain knowledge (travel, work, school) would be strategic.
Whether to produce a low priced product and gain market share or produce a high
priced product for a niche market would be a strategic decision.
2. Tactical. Tactical decisions support strategic decisions. They tend to be medium
range, medium significance, with moderate consequences.

For example: If your strategic decision were to become a forest ranger, a tactical
decision would include where to go to school and what books to read. Or if your
company decided to produce a low priced product, a tactical decision might be to
build a new factory to produce them at a low manufacturing cost.

3. Operational. These are every day decisions, used to support tactical decisions.
They are often made with little thought and are structured. Their impact is
immediate, short term, short range, and usually low cost. The consequences of a
bad operational decision will be minimal, although a series of bad or sloppy
operational decisions can cause harm. Operational decisions can be
preprogrammed, pre-made, or set out clearly in policy manuals.

For example: If your tactical decision is to read some books on forestry, your
operational decision would involve where to shop for the books. You might have a
personal policy of shopping for books at a certain store or two. Thus, the
operational decision is highly structured: "Whenever books are needed, look at
Joe's Books."

An important comment should be made here. Issues should be examined and


decisions should be made at all of these levels. If you discover that nearly all of
your thinking and decision making is taking place at the operational level, then you
are probably not doing enough strategic thinking and planning. As a result you will
lead a reactive life, responding only to the forces around you and never getting
control of your life, your direction or your goals.

Some Techniques for Decision Making


This is a list of easy, practical techniques that can be applied to simple or complex
decisions. They share the assumption that circumspect analysis is the key to
making good decisions. Many decisions are made with too little information and
too little thought, in a non-deliberate way. Think about it for a moment: how many
people do you know who commonly spend even five minutes structuring and
analyzing a decision?

Note how these techniques provide a visible, structured, orderly set of factors
involved in a decision, so that the decision maker can consider them in a thoughtful
and coherent way. The first three techniques are especially for whether-type
decisions, those involving yes/no, either/or, or two-possibility decisions.

1. T-Chart. A T-Chart is an orderly, graphic representation of alternative features


or points involved in a decision. In one form, it can be a list of positive and
negative attributes surrounding a particular choice. Drawing up such a chart
insures that both the positive and negative aspects of each direction or decision will
be taken into account.

For example, what are the pros and cons of deciding to buy a sport utility vehicle?

PRO CON

better visibility higher insurance

safer structure poorer gas mileage

can take off road more expensive maintenance


In another form, two possible choices are listed, with the good points or arguments
or effects listed for each. Suppose your company is trying to decide whether to
create its own advertising or hire an agency.

Use Outside Agency Write Ads In-House

professional work faster product

expertise of ideas better knowledge of product

media connections use same ad in flyers

To fill out this latter form, more than two choices can be included, and a list of
negatives for each choice can be added as well.

2. PMI. Edward de Bono refines the T-Chart idea into a three part structure, which
he calls PMI for plus, minus, and interesting. Here you first list all the plus or good
points of the idea, then all the minus or bad points, and finally all the interesting
points--consequences, areas of curiosity or uncertainty, or attributes that you
simply don't care to view as either good or bad at this point (consequences that
some people might view as good and others might view as bad, for example). The
"interesting" category also allows exploration of the idea or choice outside the
context of judgment--you don't have to evaluate the attribute into a positive or
negative category.
As simple as this technique seems to be, and as often as others will tell you, "Well,
of course, everyone does that all the time," this is a very powerful but much
neglected technique. Most people believe they list the pluses and minuses of a
decision before making it, but in actual practice, many people make a decision or
form an opinion before they consider the evidence in an orderly way. Only after
they make a decision do they hunt around for reasons to support it.

Considering the evidence on both (or all) sides before you commit yourself
emotionally and psychologically to a position will have a major impact on the
quality of your decision making.

3. Buriden's Ass. This method of decision making is used when two or more
equally attractive alternatives are faced. (From an old fable of an ass placed
between two equally nice bales of hay. The ass couldn't decide which bale to turn
to because they were both so attractive, and so it starved to death from indecision.)
The method is simply to list all the negative points or drawbacks about each
decision. That is, when two or more alternatives seem very desirable, we become
blinded to any drawbacks. The Buriden's Ass method simply focuses on the
drawbacks.

4. Measured Criteria. With this technique, you list the criteria you want your
decision to meet and assign points to each criterion based on its relative importance
in the decision. Then, each alternative is given a certain number of points
according to how fully it meets the criterion. For points you can use a scale of 1 to
10, 1 to 100, or any other range that makes sense to you.

In the example below, traveling by train is rated at 25 out of 30 points for the
"comfort" criterion, while the plane is ranked a little less comfortable, at 21 out of
30. Once all the alternatives have been assigned their due points for each criterion,
all the points for each alternative are added up and the alternative with the highest
total points is the one chosen. In the example below, that would be the plane.

5. Decision Matrix or Weighted Decision Table. This is a slightly more


sophisticated version of the measured criteria technique. Here a table is set up with
each criterion given a weight depending on its importance in the decision and with
each alternative given a ranking for that criterion.

Q.3 Elaborate the different stages in process of conflict?

ANS: The conflict process can be seen as comprising five stages:

(1) Potential opposition or incompatibility- The first step in the conflict process is
the presence on conditions that create opportunities for conflict to rise. These cause
or create opportunities for conflict to rise.
(2) Cognition and personalization -conflict must be perceived by the parties to it
whether or not conflict exists is a perception issue.
(3) Intentions -Intentions are decisions to act in a given way intentions intervene
between people‘s perception and emotions and their overt behavior. These
intentions are Competing, Collaborating, Avoiding, Accommodating or
Compromising.
(4) Behavior -This is a stage where conflict becomes visible. The behavior stage
includes the statements, actions and reactions made by the conflicting parties.
(5) Outcome- The action reaction interplay between the conflicting parties result in
consequences. These outcomes may be functional in that the conflict results in an
improvement in the group‘s performance, or dysfunctional in that it hinders group
performance.
Q.4 Write a note on GAS ( General Adaptation Syndrome)?

ANS: GAS or General Adaptation Syndrome is a very common medical problem


that can have very serious repercussions if left unattended. It was psychologist
Hans Selye who discovered that prolonged and excessive stress can lead to
infection, illness, disease and death. He then named this condition General
Adaptation Syndrome (GAS).

In general, there are three stages of GAS. They are Alarm, Resistance and
Exhaustion. Let's talk a bit about each stage.

Stage One: Alarm

Alarm is essentially the initiation of the "fight of flight" response which prepares
the body for life-threatening situations. Normal body activities such as the
digestive and immune functions are blunted or nearly shut down in order to supply
resources to more immediate muscular and emotional needs.

Stage Two: Resistance

Now, if we're under a constant state of alarm, we are in essence living in constant
state of stress. At this point we start becoming used to these stress levels.

The funny thing about this stage is that initially our body and immune system are
becoming more resistance to stress and disease.

However, this stage requires abnormally high levels of emotional and physical
resources. If things do not change for the better then the next stage of the GAS
process is inevitable.
Stage Three: Exhaustion

Eventually reality kicks in and our bodies are unable to maintain high levels of
stress resistance. Parts of the body literally start to break down and we become
very unwell.

To conclude this first section, please understand that Hans Selye and others experts
are convinced that out-of-control stress negatively influences a person's entire
organism. Furthermore, these authorities believe that if left untreated, run away
stress can result in disease and eventual death.

The point here is that as a hypnotist, your contribution as a stress management


consultant is far more important than most people realize. Unmanaged stress is not
just an 'inconvenience'. It is a health threaten By the way, remember to take some
of your own medicine once in a while and listen to stress reduction hypnosis CDs.
It is much easier to convince a client of the benefits of successful stress
management when you are 100% congruent because of satisfying experience.

By the way, remember to take some of your own medicine once in a while and
listen to stress reduction hypnosis CDs. It is much easier to convince a client of the
benefits of successful stress management when you are 100% congruent because of
satisfying experience.

Now, let's look at a case history that shows how to help a client suffering from
severe GAS.

When You're Client Has GAS --The Case of Bob

First of all, in order to help a client with G.A.S., they must learn how to F.A.R.T.
and B.U.R.P. so they can ultimately P.O.O.P.
Now, before a person can B.U.R.P. (Begin Utilizing Response Procedures) or
P.O.O.P. (Pursue Optimal Outcomes Persistently), they have to F.A.R.T. That is,
they have to first Formulate Appropriate Response Techniques.

This can be reduced to a simple statement. When a person starts to suffer from
stress, they must S.T.O.P. and B.E.A.N.O.

That is, they must Start To Observe Purposefully so that they can then Become
Excited And Noticeably Optimistic. If they are unable to S.T.O.P and B.E.A.N.O.,
then they will have to F.A.R.T., B.U.R.P. and most likely, P.O.O.P.

Let's look at a case history involving my client Bob. Bob has suffered for quite
some time from severe stress effects.

When he first came to our office, we realized that he was emotionally plugged up.
In other words, his feelings were so bottled up that he absolutely could not
P.O.O.P.

The reason became apparent when it was discovered that he had never before
learned how to properly F.A.R.T. and B.U.R.P. in response to stress.

I knew this was not a case where the client could simply B.E.A.N.O. because that
would not address the cause of his inability to P.O.O.P.

What he needed was an E.N.E.M.A. (Entirely New Explicit Meaning Association).

As the reader may have guessed, this was a very sensitive situation because it
involved reframing a learned response that was created in response to a family of
origin dynamic.
Apparently there was some S.O.B. (Subtle Obfuscating Behavior) responsible for
the onset of the client's difficulty.

Defeating this without running into family loyalty conflicts and their
accompanying resistance effects is quite a job that requires deft handling and
discernment.

However, since the client was extremely motivated to get past his problem and
move on we advanced a rather bold tactic.

We decided to initiate a reverse E.N.E.M.A. The client became quite excited at the
idea. This was because we explained to him that we would go back in time and
teach him as a young child how to P.O.O.P.

In summary, the client has been relieved of his GAS because after undergoing a
reverse E.N.E.M.A. he was able to B.E.L.C.H. (Begin Expurgating Lousy
Childhood History) which enabled him to successfully F.A.R.T., B.U.R.P. and
P.O.O.P.

The point to this second section is that sometimes to heal, we just need to stop
hanging onto old 'stuff' from our past because doing so makes us much more prone
to insidious medical disorders such as General Adaptation Syndrome.

One must wonder that if GAS can cause such serious consequences then, what
common diseases are actually the result of prolonged, unhealed stress?

Finally, as you can probably tell, this article was meant to be tongue-in-cheek but
the point is very serious. Helping a client reduce stress is not just a "mental
massage" that feels good in the short term. It is an essential element of a good and
happy life.
And, don't forget that the family members of your client are also benefiting from
your valuable and skilled assistance because they have longer to enjoy a healthier
and happier mom, dad or other important person.

In closing I'd like to share a quote that can point a person toward a significantly
more relaxed state of mind and being:

Q.5 Discuss the power and influence tactics?

ANS: The general objective of this study was to examine the supervisors‘
and subordinates‘ use of power and their relationships to supervisors‘ use of
influence tactics. Specifically, the purpose of this study was to examine power
congruence and its impact on influence tactics in manufacturing companies in
Malaysia. The present research differs from the previous studies by linking
power congruence between supervisors‘ and subordinates‘ power either from
self or as perceived by their subordinates or supervisors with three
dimensions of influence tactics known as, hard, soft, and rational appeal
tactics. This study is perhaps the first that tested ―congruence hypothesis‖ in
leadership framework. The objective was to gain insight into ways by which
the management of manufacturing companies might use their power to
enhance the effective use of influence tactics on their subordinates. Ten
broadly hypothesized relationships were tested in a field study with a sample
of 385 pairs of supervisors and subordinates working in 82 manufacturing
companies in Selangor/Kuala Lumpur, Penang, and Sarawak. Data were
gathered from both supervisors and their subordinates by means of
questionnaires. Methodologically, past research had been prone to common
method bias. However, this study has demonstrated to be relatively free from
this bias by collecting data from two sources. By and large, the results from
the analyses have indicated moderate support for the hypotheses. This study
is perhaps the first to generate a new set of power congruence items in which
simultaneous measurement from two perspectives-supervisors and
subordinates-were taken to examine the aspect of mutuality. The first four
hypotheses which investigate the direct relationship between supervisors or
subordinates power and influence tactics revealed that supervisors would
apply various influence tactics on their subordinates. Rational appeal tactics
has exhibited the highest mean as compared with soft and hard influence
tactics in the direct relationship between power and influence tactics. For the
indirect hypotheses, only one particular dimension of influence tactics was
found significant for each power congruence hypotheses. The results
confirmed that when both supervisors and subordinates were perceived to
have position power, the use of hard influence tactics was most apparent.
Conversely, when both of them were seen to have personal power,
supervisors would resort to the use of soft influence tactics. Inevitably, this
study provides a conceptual foundation for the effective use of influence
tactics. This study may be useful for those who are in positions of influence, to
help the supervisors and subordinates understand more clearly the bases of
their own actions, and the possible alternatives to their actions. Practically,
this research points to the fact that Malaysian managers and executives needto be
trained in the effective use of influence tactics.

Q.6 Explain the characteristics of organization Development?

ANS: Values of OD This model places human centered values above everything
else. They are the engine of its success. These values include mutual trust and
confidence, honesty and open communication, sensitivity and to the feeling and
emotions of others, shared goals, and a commitment to addressing and resolving
conflict (Buchanan and Huczynski, 1997, p.489). There are organizations who
value these human attributes above all other quick fix benefits. Stephen Robbins
(1986, p.461) expresses these human values more concisely as follows:

a) The individual should be treated with respect and dignity.


b) The organization climate should be characterized by trust, openness and
support.
c) Hierarchical authority and control are not regarded as effective mechanisms.
d) Problems and conflicts should be confronted, and not disguised or avoided.
e) People affected by change should be involved in its implementation.

These values make OD a unique model and it will be shown later why it is the
most suitable approach in resolving human afflictions and problems such as
poverty alleviation.

OD Approaches
There are at least three planned change models that have been identified by
Cummings and Worley (1997) as shown in figure II below. The first and the
second are principally different but the third one is an improved version of Action
Research methodology which has taken centre stage in the 1990s to take care of
the trend of emerging mega organizations, strategic alliances, mergers and public –
private partnerships. The improved version named the Contemporary Action
Research model is of great interest to the workshop participants as it can be used to
promote projects in both public and private organizations. Later in this paper it will
be shown how it can be used to promote Poverty Reduction Strategies and gender
mainstreaming in local government.
The distinction between Lewin‘s model and Action Research model is in the
repetitive nature of action research. Lewin‘s model is a once only intervention.
Buchanan and Huczynski, (1997) give seven steps that are followed in this
intervention.
1. Scouting – the initial stages of consultant and client exchanging ideas on the
problem and the appropriate approach.
2. Entry – contract is entered into to formalize the relationship of consultant and
client.
3. Diagnosis – information gathering to define the problem and identify causes.
4. Planning – consultant and client members jointly establish the goals of an
OD intervention and the proposed approach
5. Action – the intervention strategies are implemented.
6. Stabilization and Evaluation – the change is stabilized (freezing takes place)
and the outcomes are assessed.
7. Termination – The consultant withdraws from the assignment.

The action research model differs in several particular ways with the Lewin‘s
model. In action research the outcomes are fed back so that further improvements
and changes can be made. This distinguishes this model as a cyclical and iterative
process. The research aspect connotes a search
5
for knowledge that may be used elsewhere.

OD Toolkits
The above outlined procedure is the process through which results area achieved
but action research methodology has what are described as toolkits. These are used
to address specific areas of problem. Once diagnosis is complete and the problem
identified one or more of these kits or interventions are employed to solve the
problem. Some of these include; process consultation, change the structure, survey
feedback, team building, inter-group development, role negotiation and sensitivity
training. These are just a number but action research allows flexibility for the
consultant to formulate a package of his own interventions to suit the client‘s
6
needs. Such flexibility was illustrated through an OD consultancy in a local
government organization.

Organizational Development Interventions


The Existence of certain conditions in an organization which may be described as
the internal environment is sometimes assumed. Those who have been in an
organization for too long cease to be conscious of such an environment and
probably only newcomers notice such conditions. Different interventions are used
when a need arises to change those conditions. Some of those interventions which
are mentioned above deserve further explanation here.

Cultural Analysis
This is perhaps one of the most complex change action. Corporate culture is the
result of long term social learning and constitutes of basic assumptions, values,
norms and artifacts that have worked well in an organization. These are passed on
to succeeding generations of employees (Cummings and Worley, 1997). Some of
these may have arisen from the principles of the founder and subsequently
reinforced by succeeding top hierarchies of the organization. They may have an
emphasis on product quality, customer care or employee relations. When problems
arise the first question which would arise is how do we do things here? The answer
will inform the type of solution that emerges. Therefore the organizational culture
influences organizational strategy, performance and policies. To change
organizational culture may be a traumatic experience and will require careful
analysis and handling. Such is the trauma that was experienced by American
companies in the 1980s when they adopted the Japanese approach which was
influence by a strong organizational culture of employee participation, open
communication, Security and equality.

Process Consultation
People conceive consultation as that situation where an expert is invited to advise
an organization that is experiencing a problem. Edger Schein, (1998) has
distinguished three types of consultations as; the expertise model, the doctor –
patient model and the process consultation model. The first model assumes that the
client purchases from the consultant some expert information or service that he is
unable to provide for himself. In the doctor – patient model involves an activity
similar to sickness diagnosis. The client who suspects or feels there is something
wrong in the organization invites the doctor (consultant) to diagnose the
organization so that he can advise on what is not right. The symptoms of the
sickness may be low sales, employee instability or falling product quality. The
doctor (consultant) diagnoses, prescribes and administers the cure. In this two
models the knowledge and expertise remains with the expert and leaves the client
fully dependent on the expert for future problems. OD advocates process
consultation promoted by Schein who defines the process as:
―The creation of a relationship with the client that permits the client to
perceive, understands, and act on the process events that occur in the client‘s
internal and external environment in order to improve the situation as
defined by the client (Schein, 1998, p.20)‖.
Schein argues that both the expert and the doctor models are remedial models
while as the process consultation model is both remedial and preventive. The
purport of this model is to engage an external consultant on a flexible advisory
capacity to work with the clients members in diagnosing the problems, planning
the actions and finding the solutions together. This way the consultant helps the
organization‘s individuals to understand internal problems and build capacity to
identify appropriate problem – solving action. The consultant need not be an expert
in the problem at hand but his expertise is in facilitating a process that carries
everybody in the search for solutions. This approach follows the rationale that the
answers are with the people. So the only help required is leadership in diagnoses
and in the process through the various stages. The purpose also is to ensure that
once the consultant leaves the people have the capacity to solve the next round of
problems. In the first two models the consultant will have to be recalled each time
a new problem arises or an old one recurs.

Structure Change
This is an intervention that helps change the structure of an organization to make
the work more interesting, challenging or productive. It may involve such activities
as job enrichment, job enlargement, formation of autonomous work teams or
business re – engineering. Other actions may involve
decentralization/centralization in an organization, flattening or extending of an
organizations structure or even redesigning of focus from region to product or vice
versa.
Team Building
Team work is vital to the functioning of modern organizations. Members of teams
bring different strings to the group such as leader, investigator, motivator, finisher,
clown, coordinator, thinker, negotiator or politician. These roles are used at
different stages of production when such role play becomes essential in the groups
work. Teams take over from hierarchical systems where individuals are assumed to
know everything depending on their level in the authority ladder. This system
denies the organization the cumulative advantage of skills and strengths in
different individuals.

Role Negotiation
A misunderstanding between two individuals in an organization or group can affect
its effectiveness. This is usually caused by lack of shared awareness,
misunderstanding or lack of trust. This intervention helps to clarify individual
perceptions and mutual expectations so that differences can be identified and
reconciled or resolved.
Assignment Set- 1

Master of Business Administration - MBA Semester -1

MB0039 – Business Communication

Q.1 Explain the different types of communication with relevant examples?

ANS: Communication is a process that involves exchange of information,


thoughts, ideas and emotions. Communication is a process that involves a sender
who encodes and sends the message, which is then carried via the communication
channel to the receiver where the receiver decodes the message, processes the
information and sends an appropriate reply via the same communication channel.

Types of Communication

Communication can occur via various processes and methods and depending on
the channel used and the style of communication there can be various types of
communication.

Types of Communication Based on Communication Channels

Based on the channels used for communicating, the process of communication can
be broadly classified as verbal communication and non-verbal communication.
Verbal communication includes written and oral communication whereas the non-
verbal communication includes body language, facial expressions and visuals
diagrams or pictures used for communication.
Verbal Communication
Verbal communication is further divided into written and oral communication. The
oral communication refers to the spoken words in the communication process. Oral
communication can either be face-to-face communication or a conversation over
the phone or on the voice chat over the Internet. Spoken conversations or dialogs
are influenced by voice modulation, pitch, volume and even the speed and clarity
of speaking. The other type of verbal communication is written communication.
Written communication can be either via snail mail, or email. The effectiveness of
written communication depends on the style of writing, vocabulary used, grammar,
clarity and precision of language.

Nonverbal Communication
Non-verbal communication includes the overall body language of the person who
is speaking, which will include the body posture, the hand gestures, and overall
body movements. The facial expressions also play a major role while
communication since the expressions on a person‘s face say a lot about his/her
mood. On the other hand gestures like a handshake, a smile or a hug can
independently convey emotions. Non verbal communication can also be in the
form of pictorial representations, signboards, or even photographs, sketches and
paintings.
Types of Communication Based on Style and Purpose
Based on the style of communication, there can be two broad categories of
communication, which are formal and informal communication that have their own
set of characteristic features.
Formal Communication
Formal communication includes all the instances where communication has to
occur in a set formal format. Typically this can include all sorts of business
communication or corporate communication. The style of communication in this
form is very formal and official. Official conferences, meetings and written memos
and corporate letters are used for communication. Formal communication can also
occur between two strangers when they meet for the first time. Hence formal
communication is straightforward, official and always precise and has a stringent
and rigid tone to it.

Informal Communication
Informal communication includes instances of free unrestrained communication
between people who share a casual rapport with each other. Informal
communication requires two people to have a similar wavelength and hence occurs
between friends and family. Informal communication does not have any rigid rules
and guidelines. Informal conversations need not necessarily have boundaries of
time, place or even subjects for that matter since we all know that friendly chats
with our loved ones can simply go on and on.

Q.2 What are the general principles of writing especially business writing?

ANS: The process of good writing involves three basic steps - preparing, writing,
and editing. Practicing the following 16 principles will help you be a more
effective writer.

1. Know your objective

Think before you write. What's your goal? Make sure you fully understand the
assignment. Are you writing a one-paragraph executive summary or a five-page
report? Try answering this question: What specifically do I want the reader to
know, think, or do?

2. Make a list

Write down the ideas or points you want to cover. Why? This helps you get started
in identifying the key ideas you want to discuss.

If you have trouble getting started, try discussing your ideas with someone else.
"Kicking an idea around" often helps you clarify your objective and fine-tune what
you are trying to accomplish.

3. Organize your ideas

Just as it's difficult to find what you want in a messy, disorganized desk drawer, it's
hard to find important ideas in a poorly organized message. Here are a few ways
you can organize your ideas:

 Importance
 - Begin with the most important piece of information and then move on to
the next most important.
 Chronological order - Describe what happened first, second, third.
 Problem-Solution - Define the problem, then describe possible alternatives
or the solution you recommend.
 Question-Answer - State a question and then provide your answer.

Organize your ideas so the reader can easily follow your argument or the point you
are trying to get across.
4. Back it up

Have an opinion but back it up - support with data. There are a number of ways
you can support your ideas, including explanations, examples, facts, personal
experiences, stories, statistics, and quotations. It's best to use a combination of
approaches to develop and support your ideas.

5. Separate main ideas

Each paragraph should have one main point or idea captured in a topic sentence.
The topic sentence is normally the first sentence in the paragraph. Each paragraph
should be started by an indentation or by skipping a line.

6. Use bullets or numbers

If you are listing or discussing a number of items, use bullets or number your
points like I have done in this paper. Here's an example of using bullets.

Join the Business Club to:

 Increase sales
 Gain new marketing ideas
 Make new friends
 Give back to your profession

7. Write complete sentences

A sentence is about someone doing something - taking action. The someone may
be a manager, employee, customer, etc. The "doing something - taking action" can
include mental processes such as thinking, evaluating, and deciding, or physical
actions such as writing and talking. A good rule to practice is to have subjects
closely followed by their verbs.

8. Use short sentences

Sentences should be a maximum of 12 to 15 words in length. According to the


American Press Institute, sentences with 15 or fewer words are understood 90% of
the time. Sentences with eight or fewer words are understood 100% of the time.

9. Be precise and accurate

. Words like "large," "small," "as soon as possible," "they," "people," "teamwork,"
and "customer focus" are vague and imprecise. The reader may interpret these
words to mean something different than what you intended.

Reduce communication breakdowns by being specific and precise. Define terms as


needed. The reader may not understand certain acronyms and abbreviations.

10. Use commas appropriately

Use a comma to separate the elements in a series of three or more items:His


favorite colors are red, white, and blue.

Use a comma to set off introductory elements: After coffee and donuts, the meeting
will begin.

Use a comma to separate adjectives: That tall, distinguished, good-looking


professor teaches history.
11. Use the correct word

Here are several words that cause confusion.

 You're is a contraction for "you are" Your means possession, such as "your
coat."
 It's is a contraction for "it is." Its indicates possession.
 Their means possession/ownership-"their house." There means location.
They're is a contraction for "they are."

12. Avoid redundancies

It is a redundancy to use multiple words that mean or say the same thing. For
example, consider the following:

 Redundant - My personal beliefs… Beliefs are personal, so just state, My


beliefs...
 Redundant - I decided to paint the machine gray in color. Gray is a color,
so just state, I decided to paint the machine gray.

13. Numbers

When using numbers in the body of your paper, spell out numbers one through
nine, such as "Three men decided…" When using numbers 10 or above it's proper
to write the number, such as "The report indicated 68 customers…"
14. Have a conclusion

Would you really enjoy watching a movie or sporting event that had no
conclusion? No. The conclusion ties your points together. The reader wants to
know the final score - the bottom line message.

15. Edit your work

Read what you have written several times.

 On your first read, focus on organization and sentence structure. Shorten


long sentences. Cross out unnecessary words and phrases. Reorganize
material as needed.
 Read it again and make sure commas are used appropriately and that there is
a punctuation mark at the end of every sentence.
 Read it a third time and focus on word choice. Are there certain words that
are vague or unclear? Replace them with specific words.
 Read what you have written aloud to yourself or to a friend to see if he or
she (and you) can understand it and improve it in any way.

A significant part of good writing involves editing. Very few people can sit down
and write a perfect paragraph on their first try. It requires multiple rewrites.

Summary

You don't have to be a great writer to be successful manager/leader. However you


must be able to clearly and succinctly explain your thoughts and ideas in writing.

Strive to be simple, clear, and brief. Like any skill, "good writing" requires
practice, feedback, and ongoing improvement.
Q.3 How would you prepare yourself for an oral business presentation?

ANS: Delivering a formal presentation can be either fairly stress-free or nerve-


wrecking. Your level of comfort can depend on the size of your audience, the
critical spectators attending your presentation, or the feedback that you may
anticipate. Whatever you may find as a cause for concerns about speaking before a
group, never let it be your knowledge about what you will speak. With thorough
and effective research about your subject, you will discover that you are already
halfway prepared to address your listeners. The following steps can complete your
preparation.

1 Study your subject. You may have already been provided great information
from which you could pull. But if there are other sources, such as the Internet or
experts, use them to enhance your own insight. Doing so can also help you develop
more confidence in your speech.

2 In the comfort of your own study lab (wherever that may be), anticipate all
types of responding questions from people in your audience: challenging questions,
critical questions, crazy questions, and simple questions (the ones which are so
simple that you forgot to prepare an answer for). Equip yourself with facts and
insight accordingly. For enlightenment on people's views, I have read several
message boards and even complaint sites to help me prepare for the unexpected.

If your presentation is non-interactive, do not take that for granted. You can still be
approached with questions after your speech or after the event where you gave it.

3 Organize the notes from which you will speak. Whether typed or handwritten,
you must be able to comprehend them in order to convey them to an audience. So,
if you jot your notes down on index cards, write legibly.
Also, bind the notes - paper or cards - that you plan to use during your
presentation. Dropping loose papers or cards during your speech should not
distract an attentive listener, but it can certainly distract you, the speaker. Make
your task a tad bit easier on yourself alleviating the possibility of that problem.

4 If you decide to speak with the aid of a Power Point presentation, bear in
mind how you will insert information to be displayed. Don't expect an audience to
read lengthy sentences or any paragraph - no matter how much time they are given.

By all means, do not prepare yourself to read every word written on that Power
Point. The audience does not need to see the back of your head. They don't need to
hear your voice drift into a state of monotony, which is what can happen if you
read word-for-word from your notes.

5 Practice your presentation in private and be willing to be your own biggest


critic. Grab a tape recorder or any recording device to listen to your own speech.

6 As I expressed above, approach the podium with a solid knowledge base


about every point your will discuss.

7 Please do not imagine your audience in their underwear. You need to focus. If
eye contact with any of those listeners intimidates you, then look just past the last
row of people to land your sight on either some empty seats or the wall. There are
corners of walls and other inanimate objects where you can place your focus until
you find yourself comfortable enough to make brief eye contact with a few friendly
or neutral faces.

8 Remember that the last row of listeners need to hear your voice. Unless you
have a reliable microphone, be sure to project.
9 Do not overestimate your listeners' attention spans. Keep their interest. Give
your tone some range (logical range, that is). And wherever your subject and the
points from which you speak will allow, engage your audience with illustrations
which they can relate to.

10 Please impose neither overly technical terminologies nor acronyms on your


audience. While these expressions of intelligence seem effective, they actually
reflect a lazy effort to communicate detailed and comprehensible information to
listeners.

11 Remember that, at this point, there should be no reason to lose confidence. If


you've studied your subject, grasped a clear understanding of it, and followed the
tips above, you have the tools to conquer any sharp sensation that you may feel in
the pit of your gut going before any group.

Q.5 Distinguish between circulars and notices along with formats?

ANS: Notice - A message / information's bringing to all which will be put up in


common place

Circular- A message / information's bringing to certain group of people


belonging to the information's.

Like memos, circulars and notices are also written forms of communication
within the organization.
The difference between a circular and a notice is that circulars are
announcements that are distributed to small or selective groups of people
within the organization, whereas notices are meant for a larger group of people.

Example – If a manager wants to call a meeting of heads of departments, he will


pass around a circular only to the heads, requesting them to attend that meeting.
On the other hand, notices generally contain information or announcements that
are meant for all the employees of an organization.

Example – A list of declared holidays for a calendar year is a notice, since the
information is relevant to all employees.

A notice is therefore a legal document that has to be put up on an official notice or


bulletin board.

Let us examine another example of a circular and a notice.


Imagine that you are the President of the Student Committee in a management
college and wish to hold a meeting to plan for the Annual Management Fest of the
college. You will have to send some information to those whom you want to
involve in organizing the Fest. You may not want all the students to be involved
initially, since it may take a lot of time and there may be too many suggestions.
Instead, you may choose to invite only the committee members to discuss details
such as the date, venue, duration, how to get sponsors and so on. For this purpose,
you may send a circular only to the student committee members, requesting them
to attend the meeting. During the meeting, the date and venue may be finalized and
various smaller committees may be formed, such as a reception committee, stage
committee and so on. You may also decide to get each student to contribute a
nominal amount for the Fest
Assignment Set- 2

Q.1 As a part of top management team, how would you communicate to your
shareholders about the company’s expansion plans?

ANS:Commitment 2014 is a firm and confident ambition and it stands for


profitable organic growth. Our ambition for 2014 is to become the European
benchmark in Universal Customer-focused Banking. We intend to enhance our
leadership based on an effective product range and high-quality service, forging
close links between retail banking and related specialised businesses.

AMBITIOUS TARGETS
By 2014, we target ambitious profitability levels underpinned by sound
fundamentals: Net banking income of more than €25 billion (€20.1 billion in 2010)
and net income, Group share of €6 billion to €7 billion (€1.3 billion in 2010). Our
objective is also to have a cost to income ratio of less than 60% and a return on
equity ratio of 10% to 12%. These targets take into account the new Basel III
regulatory environment, as it is currently understood. In terms of capital adequacy,
Crédit Agricole S.A. will meet the Basel III requirements and Crédit Agricole
Group ranking among the strongest banks is confirmed.
2011-2014 STRATEGY
Our actions in the period 2011-2014 will be based on three principles. First, we
shall stimulate organic growth. To achieve this, we shall enhance growth in retail
banking, both in France and elsewhere in Europe; speed up growth in the savings
management businesses, one of our undisputed strengths; and finally, focus on
growth in investment banking and credit businesses. These businesses will
continue to grow, but selectively, because of their heavy capital consumption. The
second principle behind our strategy is that we shall enhance the Group effect by
strengthening ties between our business lines. The third principle is that we shall
act as a committed and responsible Group in dealing with stakeholders, and in
particular our shareholders.
OUR SHAREHOLDERS
Crédit Agricole S.A. has set up a comprehensive shareholder information and
communication framework. The new layout of the Shareholders‘ Club Newsletter,
―En direct‖ you discover today, is aimed at strengthening this direct link between
you and us, and is part of a broader attempt to make full and transparent
information available to you. Our dividend policy targets a payout rate of 35%
from 2011 (paid in 2012), in cash. Through Commitment 2014, we want to thank
our shareholders for their loyalty by paying an increased dividend.

Q.2 ABC Ltd. wants to communicate about its corporate image to all its
stakeholders and also to the general public. As an advisor, how do you
recommend them to do it?

ANS: Our communications and engagement strategy needs to be simple because


we are working in a complex environment – otherwise our actions become
confusing to the people we work with. The strategy therefore highlights just 20 key
steps we will take to communicate and engage with people effectively. It also
explains our simplified
approach to outreach work, including events and festivals.
Recommendation that: you agree the strategy and its associated annexes.
2 BACKGROUND
2.1 We have significantly improved the way we communicate and engage with
people over the last few years. This has been recognised through anecdotal
evidence and the recent Customer Service Excellence award. However, we need a
strategy to coordinate our activities and guide future business planning. We also
need to be able to measure our success more effectively. ‗ABC – our
communications and engagement strategy‘ (Annex 1) aims to do this.

3 POLICY CONTEXT
3.1 The adoption of a Communications and Engagement Strategy is action 3.2 in
the Business Plan July 2006-March 2009 (revised October 2007).
3.2 The strategy supports the delivery of our Business Plan in its entirety. The
messages within it explain that we must be excellent communicators, promoting
learning and inspiring people about the National Park in all that we do.

4 OPTIONS
4.1 Option 1: You agree the recommendation
Option 2: You agree the recommendation with further development of some steps
Option 3: You do not agree the recommendation

5 PROPOSALS
5.1 The Communications and Engagement Task and Finish Group was established
to develop this strategy. It includes staff from the Communications and Learning
Team and members. They undertook a SWOT analysis (Strengths, Weaknesses,
Opportunities and Threats) and agreed the simple approach recommended. They
commented on the 20 key steps ensuring that they were strategic and not too
detailed. Option 1 is recommended for approval and it is hoped that members will
be involved in the delivery and review of the strategy as it progresses.

6 BEST VALUE IMPLICATIONS


6.1 The statutory duty of best value requires organisations to consult service users
and other stakeholders about services and priorities. The duty was revised in the
Local Government and Public Involvement in Health Bill in May 2007 to engage
more with hard-to-reach groups and to secure participation of communities in
delivery of local public services. This strategy supports the delivery of this
function by focusing on engagement with partners and local people, particularly
through two of our four key
Lake District National Park Authority Agenda Item: 9
Authority: 22 May 2008
requirements for effective communications – listening and interpreting demands;
unraveling complex issues.

7 FINANCE CONSIDERATIONS
7.1 There are financial implications in the delivery of the strategy. You have
already agreed revenue growth bids (totalling £45,000) to support step 9 in the
strategy and development of Level One events and festivals which we sponsor
(Annex B). You have also agreed existing budgets to support ongoing work led by
the Communications and Learning Team. Agreement of any additional budgetary
requirements to deliver all steps in the strategy will form part of the corporate
planning and performance cycle.
8 RISK
8.1 Each step in the strategy has a different degree of risk which will be managed
through respective service plans. Overall, if we do not develop our
communications and engagement activities, there is a significant risk that we lose
our Customer Service Excellence standard.

9 LEGAL CONSIDERATIONS
9.1 Effective communication is key to performance of our statutory objectives. The
proposed strategy will assist with good governance by improving communication
within the Authority and will help clarify relationships with partners.

10 HUMAN RESOURCES
10.1 The Communications and Learning Team, including its Outreach Unit, are
part of the established structure and will take a clear coordinating role in delivering
the strategy.
The strategy also makes it clear that new approaches to communications and
engagement should be a part of existing roles across the organisation. This will be
dealt with through Service Planning and Personal Development Reviews.

11 DIVERSITY IMPLICATIONS
11.1 This strategy is of high importance to diversity. In the past, our approach to
targeting groups may have had an adverse impact on particular groups already
using our services. The strategy explains that we will now base our actions on the
needs of different groups and clear evidence. Annex A (‗Reaching Out‘) explains
how we will not target people, but work with focus groups to identify needs over
time. We want toreach out to everyone and use the National Park to build links
between different cultures.
12 SUSTAINABILITY
12.1 The strategy will contribute to the promotion of learning and inspiring people
about all aspects of sustainability which is at the heart of the Vision for the
National Park.
Author/Post Bob Sutcliffe, Head of Plans and Communications
Date Written 2 May 2008

Q.3 What is oral business communication? Explain its benefits to the


organisation and to the individual employee.
ANS: According to a 2005 study published in the Journal of Employment
Counseling, oral communication skills are being increasingly sought after by
employers. When surveying over 100 successful businesses, researchers found that
more and more employers are emphasizing the development of good speaking
skills in their employees. With this in mind, the concept of oral
communication is an important idea to study and understand in the context of
business.

Presentations:
One form of oral communication in a business setting is a presentation.
Presentations are usually an organized conveyance of information to a group of
people. Stylistically, they tend to be far more formal than informal, and rely more
heavily on data and facts than they do analysis.
Presentations are sometimes more persuasive in nature, like a pitch for an ad
campaign, but tend to be informative more often, such as an employee briefing or a
report on quarterly earnings.
Presentations may include some dialog after the sender of the message has finished
their speech, but they are, by and large, much more monologue reliant. This makes
it important for the speaker to anticipate possible objections to the message and
address them in the actual speech.

Client Interaction:
Another form of oral communication in business encompasses interaction with
clients.
Depending on the level of connection between the employee and the client, the
communication in these interactions can range from incredibly formal to informal
and casual. These interactions usually include a combination of data and analysis,
and will be more persuasive than informative in nature, as the employee is trying to
encourage continued and expanded business with the client. Because of the nature
of these interactions, the communication is definitely a dialog, making listening
skills incredibly important.

Interoffice Interaction:
Oral communication in the office can be referred to as interoffice interaction. This
is comprised of conversations with superiors, subordinates and co-workers.
Depending on the levels of power separation between the individuals engaging in
conversation, the communication will fluctuate between formal and informal,
though it should always remain professional. Conversations in this
context may reference data, but will be much more analysis heavy, and will be a
dialog by nature.

Benefits:
Oral communication in business provides a variety of benefits. First, oral
communication is accompanied by nonverbal signifiers, which provides context
that can enhance understanding in the communication process. Posture, facial
expressions, and habitual movements may provide
clues as to an individuals feelings about the ideas being discussed. Even in
telephone conversations, pitch, rate, volume and tone of the respective speakers
can help in understanding sentiments.
Oral communication also provides a springboard for relational development.
Unlike with email, memos and chat functions, which tend to take a task-oriented
approach to communication, the immediacy involved in oral communication
allows for instant feedback and a more relational approach. This is important, as
strong relationships in business often lead to more profitable and productive
cooperation.

Q.4. Give short notes on communication network in the organisation?

ANS: Networks are another aspect of direction and flow of communication.


Bavelas has shown that communication patterns, or networks, influence groups in
several important ways.
Communication networks may affect the group's completion of the assigned task
on time, the position of the de facto leader in the group, or they may affect the
group members' satisfaction from occupying certain positions in the network.
Although these findings are based on laboratory experiments, they have important
implications for the dynamics of communication in formal organizations.

There are several patterns of communication:


"Chain",
"Wheel",
"Star",
"All-Channel" network,
"Circle".

The Chain can readily be seen to represent the hierarchical pattern that
characterizes strictly formal information flow, "from the top down," in military and
some types of business organizations. The Wheel can be compared with a typical
autocratic organization, meaning oneman rule and limited employee participation.
The Star is similar to the basic formal structure of many organizations. The All-
Channel network, which is an elaboration of Bavelas's Circle used by Guetzkow, is
analogous to the free-flow of communication in a group that encourages all of
its members to become involved in group decision processes. The All-Channel
network may also be compared to some of the informal communication networks.
If it's assumed that messages may move in both directions between stations in the
networks, it is easy to see that some individuals occupy key positions with regard
to the number of messages they handle and the degree to which they exercise
control over the flow of information. For example, the person represented by the
central dot in the "Star" handles all messages in thegroup. In contrast, individuals
who occupy stations at the edges of the pattern handle fewer
messages and have little or no control over the flow of information.These
"peripheral" individuals can communicate with only one or two other persons and
must depend entirely on others to relay their messages if they wish to extend their
range.
In reporting the results of experiments involving the Circle, Wheel, and Star
configurations, Bavelas came to the following tentative conclusions. In patterns
with positions located centrally, such as the Wheel and the Star, an organization
quickly develops around the people occupying these central positions. In such
patterns, the organization is more stable and errors in performance are lower than
in patterns having a lower degree of centrality, such as the Circle.
However, he also found that the morale of members in high centrality patterns is
relatively low. Bavelas speculated that this lower morale could, in the long run,
lower the accuracy and speed of such networks.
In problem solving requiring the pooling of data and judgments, or "insight,"
Bavelas suggested that the ability to evaluate partial results, to look at alternatives,
and to restructure problems fell off rapidly when one person was able to assume a
more central (that is, more controlling) position in the information flow. For
example, insight into a problem requiring change would be
less in the Wheel and the Star than in the Circle or the Chain because of the
"bottlenecking" effect of data control by central members.
It may be concluded from these laboratory results that the structure of
communications within an organization will have a significant influence on the
accuracy of decisions, the speed with which they can be reached, and the
satisfaction of the people involved. Consequently, in networks in
which the responsibility for initiating and passing along messages is shared more
evenly among the members, the better the group's morale in the long run.

Q. 5 What are the different types of business letters? Explain with example.

ANS: Business letter is an old form of official correspondence. A business letter is


written by an individual to an organization or an organization to another
organization. Business letters are written for various purposes. One writes a letter
to enquire information, apply for a job, acknowledge someone's work, and
appreciate one's job done, etc. As the motive of writing the letter is different, the
style of the letter changes and you get different types of business letters. The
various types of business letters are used by different people to serve their purpose
of sending the message across.

Let's take look at the most common types of business letters:

Acknowledgement Letter : This type of letter is written when you want to


acknowledge some one for his help or support when you were in trouble. The letter
can be used to just say thanks for something you have received from some one,
which is of great help to you.

Apology Letter : An apology letter is written for a failure in delivering the desired
results. If the person has taken up a task and he fails to meet the target then he
apologizes and asks for an opportunity to improve in this type of letter.

Appreciation Letter : An appreciation letter is written to appreciate some one's


work in the organization. This type of letter is written by a superior to his junior.
An organization can also write an appreciation letter to other organization,
thanking the client for doing business with them.

Complaint Letter : A complaint letter is written to show one that an error has
occurred and that needs to be corrected as soon as possible. The letter can be used
as a document that was used for warning the reader.

Inquiry Letter : The letter of inquiry is written to inquire about a product or


service. If you have ordered a product and yet not received it then you can write a
letter to inquire when you will be receiving it.
Order Letter : This letter is as the name suggests is used for ordering products.
This letter can be used as a legal document to show the transaction between the
customer and vendor.

Letter of Recommendation : This type of letter is written to recommend a person


for a job position. The letter states the positive aspects of the applicant's
personality and how he/she would be an asset for the organization. Letter of
recommendation is even used for promoting a person in the organization.
Assignment Set- 1
Master of Business Administration –
MBA Semester -1

MB0040 – STATISTICS FOR MANAGEMENT

Q. 1 What is the difference between a qualitative and quantitative variable?


ANS: Scientific experiments will normally have three types of variables;
controlled, independent and dependent. Variables are a condition or factor that is
used in testing a hypothesis and generating a conclusion. These three types of
variables can also be quantitative or qualitative in nature.

Qualitative:

By definition something that is qualitative concerns or describes a quality. A


qualitative variable is a descriptive. Qualitative variable are sometimes referred to
as categorical. The variable may be colors in the light spectrum or a comparison
between red and green grapes. Qualitative variables can influence the outcome of
an experiment or research because they can influence other factors or parameters.
Qualitative variables are frequently used in social research. Qualitative research is
considered to be inductive.

Quantitative:

By definition something that is quantitative can be expressed as a quantity or


number. Quantitative variables are something that can be measured. Quantitative
variables are numerical. A quantitative variable can be a percentage of something,
a number of units or any other measurement.
Temperature is a quantitative value or variable by the number of degrees. Speed,
area population, voltage and time are all examples of quantitative variables that can
be measured. Quantitative variables are most often considered to be deductive in
nature.

Deduction and induction in experimentation and research:

Deduction works from a general idea to a specific idea. Deductive research starts
with a theory, forms a hypothesis, gathers observations and then confirms or
disproves the original thought.

Induction works in the reverse. Inductive experimentation will start with an


observation and then look for patterns in the observation. Once patterns form a
hypothesis is developed. The hypothesis is then tested for a resulting theory.

The best results in experimentation come from having only one independent
variable. The controlled variable is something that does not change and must
remain constant. The independent variable is the variable that is changed by the
researcher. The dependent value is the variable that changes due to the independent
variable.

An example of quantitative variables in an experiment would be testing the change


in speed on a turntable as additional weight is applied. The turntable itself is the
controlled variable. The experimenter will only use one. The independent
quantitative variable is the amount of weight applied for each measurement. The
dependent quantitative variable is the resulting speed that is measured.

An example of a qualitative variable in testing would be the drying time require for
red and green grapes at a constant temperature. The outcome, or dependent
variable, of time is measured and therefore quantitative. The controlled variable
being used is temperature, also quantitative. The independent variable is
qualitative, the difference between red and green grapes. In this particular example
the weight of each grape, a quantitative variable would also need to be consistent
or controlled.

Q 2. a) Explain the steps involved in planning of a statistical survey?


ANS: I've explained the fundamentals of creating and executing surveys. We've
explored how to construct questions so they yield accurate, useful data. We've
discussed how to boost your response rate, thereby squeezing more value from
your efforts. And we've talked briefly about the various strategies you can employ
to solicit participation from your respondents. But, everything begins with the
initial plan; you shouldn't approach it haphazardly. In this article, I'll describe the 5
steps that you and your design team should follow to plan a successful survey
execution.

Step #1: Identify Your Objective

This first step is more involved than you might think and is the foundation upon
which the other steps depend. It's not enough to say, "I need to know if customers
are satisfied" or "I want to measure employee morale." You should know in
advance the precise reasons you want to collect data, what you hope to learn from
the information, and the actions you intend to take once you and your team have
reviewed the responses. Unless you have already identified a specific reason for
conducting your survey, including how it will add value to your business or
organization, you shouldn't move forward.
Step #2: Determine How Much It Will Cost

Design and deployment can be expensive depending upon its scope, the
information you hope to collect, and your population size. Some of the costs aren't
immediately obvious. For example, if you're using employees to approach
participants, you'll need to pay for their time. If you're conducting a survey by
mail, you'll have to pay postage. Analyzing the data also carries a cost. Plan
conservatively to ensure you have the budget to see the project through its
completion.

Step #3: Plan The Logistics

You'll need to choose your participant pool, figure out how to approach them, and
determine how much time your project will require. Don't take any of these items
lightly. Your selection of the people you choose to approach will depend upon the
type of information you hope to collect. For example, surveying single men
between ages 30 and 39 will yield different data than married couples over 60.
Your strategy for approaching them can vary between phone, email, in person
interviews, postal mail, etc. The time you'll need to complete your survey will be
based largely upon these factors.

Step #4: Define Your Resources

Beyond your budget, time constraints, and other limitations; you'll need to identify
resources (both internal and external) upon which you'll rely for expertise and
assistance. For example, if you're measuring employee satisfaction, you'll want to
enlist the help of your organization's human resources department. If you're
outsourcing the design of your survey, you'll need to identify and interview a
number of experienced agencies. During this step, you should also determine the
internal personnel who will need to see your team's analysis of the data (i.e.
president, CFO, accounting department, etc.).

Step #5: Map The Steps To Completion

Before you execute your survey, take the time to create a chronological timetable
of the project. It should detail each step in the process, including who is
responsible for collecting the information needed to complete each step and the
specific tasks that are involved along the way. Without creating this plan ahead of
time, you risk wasting finite resources as a result of confusion and lack of
accountability.

Successful Deployment

Creating and executing effective surveys that yield useful data require a lot of time
and upfront planning. If you approach the task methodically and follow the 5 steps
we've covered above, you'll dramatically improve the likelihood of success.
Remember your objective: to collect accurate information that you can take action
upon for a predefined benefit to your organization. In an upcoming article, we'll
explore the deployment in more detail.

b) What are the merits & Demerits of Direct personal observation and
Indirect Oral Interview?
ANS: In the direct personal observation method, the investigator collects data by
having direct contact with units of investigation.The accuracy of data depends
upon the ability, training and attitude of the investigator.

The direct personal observation method is suitable where -


- The scope of investigation is narrow
- Investigation is confidential and requires personal attention of the investigator
- Accuracy of data is important

Merits and demerits of direct personal observation -

Merits:
1) We get original data.
2) we get more accurate and reliable data.
3) Satisfactory information can be extracted by the investigator through indirect
questions.
4) Data are homogeneous and comparable.
5) Additional information can be gathered.
6) Misinterpretation of questions can be avoided.

Demerits:
1) It is time consuming and costs more.

Indirect oral interview is used when area to be covered is large. The data is
collected from a third party or witness or head of institution. This method is
generally used by police department.
Merits and demerits of indirect oral interview -

Merits:
1) It is economical in terms of time, cost and manpower.
2) Confidential information can be collected.
3) Information is likely to be unbiased and reliable.

Demerits:
1) The degree of accuracy of information is less.

Q 3. a) What is the main difference between correlation analysis and


regression analysis?
ANS: (1) The correlation answers the STRENGTH of linear association between
paired variables, say X and Y. On the other hand, the regression tells us the FORM
of linear association that best predicts Y from the values of X.

(2a) Correlation is calculated whenever:


* both X and Y is measured in each subject and quantify how much they are
linearly associated.
* in particular the Pearson's product moment correlation coefficient is used when
the assumption of both X and Y are sampled from normally-distributed populations
are satisfied
* or the Spearman's moment order correlation coefficient is used if the assumption
of normality is not satisfied.
* correlation is not used when the variables are manipulated, for example, in
experiments.
(2b) Linear regression is used whenever:
* at least one of the independent variables (Xi's) is to predict the dependent
variable Y. Note: Some of the Xi's are dummy variables, i.e. Xi = 0 or 1, which are
used to code some nominal variables.
* if one manipulates the X variable, e.g. in an experiment.

(3) Linear regression are not symmetric in terms of X and Y. That is interchanging
X and Y will give a different regression model (i.e. X in terms of Y) against the
original Y in terms of X.
On the other hand, if you interchange variables X and Y in the calculation of
correlation coefficient you will get the same value of this correlation coefficient.

(4) The "best" linear regression model is obtained by selecting the variables (X's)
with at least strong correlation to Y, i.e. >= 0.80 or <= -0.80

(5) The same underlying distribution is assumed for all variables in linear
regression. Thus, linear regression will underestimate the correlation of the
independent and dependent when they (X's and Y) come from different underlying
distributions.

b) In a multiple regression model with 12 independent variables, what are the


degrees of freedom for error? Explain?

ANS: Consider a random sample of n observations (xi1, xi2, . . . . , xip, yi), i = 1, 2, . .


. , n.

The p + 1 random variables are assumed to satisfy the linear model

yi 0 1xi1 2xi2 pxip + ui i = 1, 2, . . . , n


where ui are values of an unobserved error term, u, and. the unknown parameters
are constants.

Assumptions

 The error terms ui are mutually independent and identically distributed, with
mean = 0 and constant variances

E [ui] = 0 V [ui] =

 This is so, because the observations y1, y2, . . . ,yn are a random sample, they
are mutually independent and hence the error terms are also mutually
independent
 The distribution of the error term is independent of the joint distribution of x
i, x 2, . . . , x p
 0 1 2 p are constants.

Equations relating the n observations can be written as:

0 1 p can be estimated using the least squares


procedure, which minimizes the sum of squares of errors.

Minimizing the sum of squares leads to the following equations, from which the
Geometrical Representation

The problem of multiple regression can be geometrically represented as follows.


We can visualize that n observations (xi1, xi2, …..xip, yi) i = 1, 2, ….n are
represented as points in a (p+1) - dimensional space. The regression problem is to
determine the possible hyper-planes in the p – dimensional space, which will be
the best- fit. We use the least squares criterion and locate the hyper-plane that
minimizes the sum of squares of the errors, i.e., the distances from the points
around the plane (observations) and the point on the plane.

(i.e. the estimate ŷ).

ŷ = a+b1x1+b2x2+…+bpxp

Standard error of the estimate

Se =

where yi = the sample value of the dependent variable

ŷi = corresponding value estimated from the regression equation


n = number observations

p = number of predictors or independent variable

The denominator of the equation indicates that in multiple regression with p


independent variables, the standard error has n-p-1 degrees of freedom. This
happens because the degrees of freedom are reduced from n by p+1 numerical
constants a, b1, b2, …..bp, that have been estimated from the sample.

Fit of the regression model

The fit of the multiple regression model can be assessed by the Coefficient of
Multiple determination, which is a fraction that represents the proportion of total
variation of y that is explained by the regression plane.

Sum of squares due to error


SSE =

Sum of squares due to regression

SSR =

Total sum of squares

SST =

Obviously,

SST = SSR + SSE

The ratio SSR/SST represents the proportion of the total variation in y explained by
the regression model. This ratio, denoted by R2, is called the coefficient of multiple
determination. R2 is sensitive to the magnitudes of n and p in small samples. If p is
large relative to n, the model tends to fit the data very well. In the extreme case, if
n = p+1, the model would exactly fit the data.

A better goodness of fit measure is the adjusted R2, which is computed as follows:

Adjusted R2= 1 – ( ) (1-R2)

=1-
Statistical inferences for the model

The overall goodness of fit of the regression model (i.e. whether the regression
model is at all helpful in predicting the values of y can be evaluated, using an F-
test in the format of analysis of variance.

Under the null hypothesis: Ho: β1 = β2 = ... = βp = 0, the statistic

has an F-distribution with p and n--1 degrees of freedom

ANOVA Table for Multiple Regression

Source of Sum of Degrees Mean F ratio


Variation Squares of Squares
freedom

Regression SSR p MSR MSR/MSE

Error SSE (n-p-1) MSE

Total SST (n-1)

Whether a particular variable contributes significantly to the regression equation


can be tested as follows: For any specific variable xi, we can test the null
hypothesis Ho: βi = 0, by computing the statistic

t=
and performing a one or two tailed t-test with n-p-1 degrees of freedom.

Standardized regression coefficients

The magnitude of the regression coefficients depends upon the scales of


measurement used for the dependent variable y and the explanatory variables
included in the regression equation. Unstandardized regression coefficients cannot
be compared directly because of differing units of measurements and different
variances of the x variables. It is therefore necessary to standardize the variables
for meaningful comparisons.

The estimated model

ŷi = bo+b1xi1+b2xi2+….bpxip

can be written as:

The expressions in the parentheses are standardized variables; b’s; are


unstandardized regression coefficients and s1, s2, …sp are the standard deviations of
variables x1, x2, ….xp and sx is the standard deviation of variable y. The coefficients
(bisi)/sy, j=1,2,…,p are called standardized regression coefficients. The
standardized regression coefficient measures the impact of a unit change in the
standardized value of xi on the standardized value of y. The larger the magnitude of
standardized bi, the more xi contributes to the prediction of y. However, the
regression equation itself should be reported in terms of the unstandardized
regression coefficients so that prediction of y can be made directly from the x
variables.
Multiple Correlation

Multiple correlation coefficient, R, is a measure of the strength of the linear


relationship between y and the set of variables x1, x2, …xp. It is the highest possible
simple correlation between y and any linear combination of x1,x2,….,xp. This
property explains that the computed value of R is never negative. In this sense, the
least squares regression plane maximizes the correlation between the x variables
and the dependent variable y. Hence, it represents a measure of how well the
regression equation fits the data. When the value of the multiple correlation R is
close to zero, the regression equation barely predicts y better than sheer chance. A
value of R close to 1 indicates a very good fit.

Partial Correlation

A useful approach to study the relationship between two variables x and y in the
presence of a third variable z is to determine the correlation between x and y after
controlling the effect of z. This correlation is called partial correlation. Partial
correlation is the correlation of two variables while controlling for a third or more
other variables. For example r12.34 is the correlation of variables 1 and 2,
controlling for variables 3 and 4. If partial correlation r12.34 is equal to uncontrolled
correlation r12 , it implies that the control variables have no effect on the
relationship between variables 1 and 2.. If partial correlation is nearly equal to
zero, it implies that the correlation between original variable is spurious.

Partial correlation coefficient is a measure of the linear association between two


variables after adjusting for the linear effect of a group of other variables. If the
number of other variables is equal to 1, the partial correlation coefficient is called
the first order coefficient. If the number of other variables is equal to 2, the partial
correlation coefficient is called the second order coefficient, and so on.

First order Partial Correlation

The first order partial correlation between xi and xj holding constant xl is computed
by the following formula

rij.l =

where rij, ril and rjl are zero order (Pearson‘s r coefficient)

Second order Partial Correlation

Correlation between xi and xj holding constant xl and xm is computed by the


following formula:

rij.lm =

where rij, rim.l, rjm.l are first order partial correlation coefficients.

Statistical significance of partial correlation coefficients can be tested by using a


test statistic similar to the one for simple correlation coefficient.

t=

where q is the number of variables held constant. The value of t is compared with
tabulated t for n-q-2 degrees of freedom.
Multicollinearity

In practice, the problem of multicollinearity occurs when some of the x variables


are highly correlated. Multicollinearity can have significant impact on the quality
and stability of the fitted regression model. A common approach to
multicollinearity problem is to omit explanatory variables. For example if x1 and x2
are highly correlated (say correlation is greater than 0.9), then the simplest
approach would be to use only one of them, since one variable conveys essentially
all the information in the other variable.

The simplest method for detecting multicollinearity is the correlation matrix, which
can be used to detect if there are large correlations between pairs of explanatory
variables.

When more subtle patterns of correlation coefficients exist, the determinant of the
correlation matrix computed by IDAMS can be used to detect multicollinearity.
The determinant of the correlation matrix represents as a single number the
generalized variance in the set of predictor variables, and varies from 0 to 1. The
value of the determinant near zero indicates that some or all explanatory variables
are highly correlated. The value of the determinant equal to zero indicates a
singular matrix, which indicates that at least one of the predictors is a linear
function of one or more other predictors.

Another approach is to compute the ‗tolerance‘ associated with a predictor. The


tolerance of xi is defined as 1 minus the squared multiple correlation between that
xi and the remaining x variables. When tolerance is small, say less than 0.01, then it
would be expedient to discard the variable with the smallest tolerance. The inverse
of the tolerance is called the variance inflation factor (VIF).
Stepwise Regression

Stepwise regression is a sequential process for fitting the least squares model,
where at each step a single explanatory variable is either added to or removed from
the model in the next fit.

The most commonly used criterion for the addition or deletion of variables in
stepwise regression is based on partial F-statistic:

The suffix ‗Full‘ refers to the larger model with p explanatory variables, whereas
the suffix ‗Reduced‘ refers to the reduced model with ( ) explanatory
variables.

Forward selection

Forward selection procedure begins with no explanatory variable in the model and
sequentially adds a variable according to the criterion of partial F- statistic. At each
step, a variable is added, whose partial F- statistic yields the smallest p - value.
Variables are entered as long as the partial F-statistic p-value remains below a
specific maximum value (PIN). The procedure stops when the addition of any of
the remaining variables yields a partial p-value > PIN. This procedure has two
limitations. Some of the variables never get into the model and hence their
importance is never determined. Another limitation is that a variable once included
in the model remains there throughout the process, even if it loses its stated
significance, after the inclusion of other variable(s).
Backward elimination

The backward elimination procedure begins with all the variables in the model and
proceeds by eliminating the least useful variable at a time. A variable, whose
partial F p-value is greater than a prescribed value, POUT, is the least useful
variable and is therefore removed from the regression model. The process
continues, until no variable can be removed according to the elimination criterion.

Stepwise procedure

The stepwise procedure is a modified forward selection method which later in the
process permits the elimination of variables that become statistically non-
significant. At each step of the process, the p-values are computed for all variables
in the model. If the largest of these p-values > POUT, then that variable is
eliminated. After the included variables have been examined for exclusion, the
excluded variables are re-examined for inclusion. At each step of the process, there
can be at the most one exclusion, followed by one inclusion. It is necessary that
PIN POUT to avoid infinite cycling of the process.

Regression with Qualitative Explanatory Variables

Sometimes, explanatory variables for inclusion in a regression model are not


interval scale; they may be nominal or ordinal variables. Such variables can be
used in the regression model by creating ‗dummy‘ (or indicator) variables.

Dichotomous Variables

Dichotomous variables do not cause the regression variables to lose any of their
properties. Since they have two categories, they manage to ‗trick‘ least squares,
while entering into the regression equation as interval scale variables with just two
categories.

Consider for example, the relationship between income and gender

y = a + bx

where

y = income of an individual, and

x = a dichotomous variable, coded as

0 if female

1 if otherwise

The estimated value of y is

ŷ =a if x = 0

ŷ=a+b if x = 1

Since our best estimate for a given sample is the sample mean, a is estimated as the
average income for females and a+b is estimated as average income for males. The
regression coefficient b is therefore

male – female

In effect, females are considered as the reference group and males‘ income is
measured by how much it differs from females‘ income.
Polytomous Variables

Consider, for example, the relationship between the time spent by an academic
scientist on teaching and his rank.

y = a+bx

where

y is the percentage of work time spent on teaching

x is a polytomous variable ‗rank‘ with three modalities:

1 = Professor

2 = Reader

3 = Lecturer

We create two dummy variables:

X1 = 1 if rank = Professor

0 if otherwise

X2 = 1 if rank = Reader

0 if otherwise

Note that we have created two dummy variables to represent a trichotomous


variable. If we create a third dummy variable X3 (score 1; if rank = Lecturer, and 0
otherwise), the parameters of the regression equation cannot be estimated uniquely.
This is because if the score of any respondent on X1 and X2 is known, it would
always be possible to predict his score on X3. For example if a respondent has score
0 on X1 (not Professor) and 0 on X2 (not Reader), then the respondent is certainly a
Lecturer (i.e., score 1 on X3). This represents a situation of perfect
multicollinearity. Hence the general rule for creating dummy variables is: Number
of dummy variables = Number of modalities minus 1.

Statistical significance of regression coefficients and Multiple R2 is determined in


the same way as for interval scale explanatory variables.

Q 5. a) Discuss what is meant by Quality control and quality improvement?

ANS: Quality is a much more complicated term than it appears. Dictionary


definitions are usually inadequate in helping a quality professional understand the
concept. It seems that every quality expert defines quality is a somewhat different
way. There are a variety of perspectives that can be taken in defining quality (e.g.
customer's perspective, specification-based perspective). Are there commonalities
among these definitions? Is any one definition "more correct" than the others? Is
one quality expert "right" and the others "wrong"? Quality professionals
constantly debate this question. The editors of Quality Digest say that defining the
word "quality" is "no simple endeavor." They asked, in their December 1999
issue, for readers to send them their definitions of quality to be gathered and posted
on Quality Digest Online.

A modern definition of quality derives from Juran's "fitness for intended use."
This definition basically says that quality is "meeting or exceeding customer
expectations." Deming states that the customer's definition of quality is the only
one that matters.
Quality assurance (QA) is a broad concept that focuses on the entire quality
system including suppliers and ultimate consumers of the product or service. It
includes all activities designed to produce products and services of appropriate
quality.

According to ASQ, QA includes all those planned or systematic actions necessary


to provide adequate confidence that a product or service will satisfy given needs.
[Source: ASQ Statistics Division, Glossary & Tables for Statistical Quality
Control, 1983].

Quality control (QC) has a narrower focus than quality assurance. Quality control
focuses on the process of producing the product or service with the intent of
eliminating problems that might result in defects.

According to ASQ, QC includes the operational techniques and the activities


which sustain a quality of product or service that will satisfy given needs; also the
use of such techniques and activities. [Source: ASQ Statistics Division, Glossary
& Tables for Statistical Quality Control, 1983].

Quality management is the totality of functions involved in the determination and


achievement of quality (includes quality assurance and quality control). [Source:
ASQ Statistics Division, Glossary & Tables for Statistical Quality Control, 1983].

External customers usually come to mind first. These are the people outside our
organization who receive our goods and services. But even here there is some
confusion. If we sell our products to a wholesaler, is he our only customer? How
about the retailer and the ultimate consumer?
Internal customers are often forgotten or taken for granted. In an assembly line
operation, the next station downstream from ours is an internal customer for our
work. The Purchasing Dept. who receives a control report from the Accounting
Dept. is the Accounting Dept's. internal customer. Second grade teachers are
internal customers for first grade teachers.

Now, let's discuss meeting or exceeding customer expectations. Meeting customer


expectations results in a satisfied customer. But where is the competitive
advantage in that? Have you eaten in a restaurant in the past month? If so, did you
select a restaurant that you expected would dissatisfy you? Probably not. You
selected from a list of restaurants that you expected would satisfy you. So,
satisfying customers merely keeps you in the game. Delighting customers
(exceeding customer expectations) is where competitive advantage can be found.
Restaurants that deliver larger than expected portions or lower than expected prices
or better than expected service or better than expected ambiance (order winners)
have a competitive advantage over restaurants that simply satisfy customers
(possess only order qualifiers).

A number of scholars in the quality field have developed lists of dimensions that
define quality for a product and/or a service. David Garvin developed a list of 8
dimensions of product quality. Evans and Lindsay provide a list of 8 dimensions
of service quality. These are general lists and serve as good starting points. But,
current research indicates that in terms of service quality, the dimensions are
different for different industries. So Evans and Lindsay's list may not apply
equally well to, for example, health care services and food services. Parasuraman,
et. al. developed a general list of 5 service dimensions that they tested in 4 types of
service industry, but the applicability of these dimensions in other industries is
unknown.
Developing a list of quality dimensions for a specific service industry requires
determining what is important to customers. Methodologies which are appropriate
for this would include focus groups and surveys. The quality dimensions for
hospitals (KQCAH Scale) that are shown in the Lesson 1 Presentation were
developed using focus groups conducted with recently discharged patients and
their families, and with hospital personnel. Knowledge of these dimensions
facilitates the measurement of patient satisfaction by hospitals. Hospitals know
that they are measuring dimensions that are important to patients.

b) What are the limitations of a quality control charts?

ANS: The limitations of a quality control charts:

The quality control chart is based on the research of Villefredo Pareto. He found
that approximately 80 percent of all wealth of Italian cities he researched was held
by only 20 percent of the families. The Pareto principle has been found to apply in
other areas, from economics to quality control. Pareto charts have several
disadvantages, however.

Easy to Make but Difficult to Troubleshoot

percent of issues that cause the majority of problems in order to have the greatest
impact. However, one of the disadvantages of Pareto charts is that they provide no
insight on the root causes. For example, a Pareto chart will demonstrate that half of
all problems occur in shipping and receiving. Failure Modes Effect Analysis,
Statistical Process Control charts, run charts and cause-and-effect charts are
needed to determine the most basic reasons that the major issues identified by the
Pareto chart are occurring.
Multiple Pareto Charts May Be Needed

Pareto charts can show where the major problems are occurring.

6. a) Suggest a more suitable average in each of the following cases:

(i) Average size of ready-made garments.

ANS: (i) Average size of ready-made garments: Arithmetic mean will be used
because it is

continuous and additive in nature.

(ii) Average marks of a student.

ANS: Average marks of a student. : Arithmetic mean will be used because it is


the data are in the interval and the distribution is symmetrical.

b) State the nature of symmetry in the following cases:

(i) When median is greater than mean?

ANS: (i) When median is greater than mean : the data are "skewed to the left",
with a

long tail of low scores pulling the mean down more than the median.
Assignment Set- 2

Q 1. What are the characteristics of a good measure of central tendency?

ANS: Characteristics of a Good Average

(i) It should be rigidly defined. If an average is left to the estimation of an observer


and if it is not a definite and fixed value it cannot be representative of a series. The
bias of the investigator in such cases would considerably affect the value of the
average. If the average is rigidly defined; this instability in its value would be no
more, and it would always be a definite figure,

(ii) It should be based on all the observations of the series. If some of the items of
the series are not taken into account in its Calculation the average cannot be said to
be a representative one. As we shall see later on there are some averages which do
not take into account all the values of a group and to this extent they are not
satisfactory averages.

(iii) It should be capable of further algebraic treatment. If an average dose not


possess this quality, its use is bound to be very limited. It will not be possible to
calculate, say, the combined average of two or more series from their individual
averages; further it will not be possible to study the average relationship of various
parts of a variable if it is expressed as the sum of two or more variables. Many
other similar studies would not be possible if the average is not capable of further
algebraic treatment.

(iv) It should be easy to calculate and simple to follow. If the calculation of the
average involves tedious mathematical processes it will not be readily understood
and its use will be confined only to a limited number of persons. It can never be a
popular average. As such, one of the qualities of a good average is that it should
not be too abstract or mathematical and there should be no difficulty in its
calculation. Further, the properties of the average should be such that they can be
easily understood by persons of ordinary intelligence.

(v) It should not be affected by fluctuations of sampling. If two independent


sample studies are made in any particular field, the averages thus obtained, should
not materially differ from each other. No doubt, when two separate enquires are
made, there is bound to be a difference, in the average values calculated but in
some cases this difference would be great while in others comparatively less.
These averages in which this difference, which is technically called "fluctuation of
sampling" is less, are considered better than those in which its difference is more.

One more thing to be remembered about averages is that the items whose average
is being calculated should form a homogenous group. It is absurd to talk about the
average of a man's height and his weight. If the data from which an average is
being calculated are not homogeneous, misleading conclusions are likely to be
drawn. To find out the average production of cotton cloth per mill, if big and small
mills are not separated the average would be unrepresentative. Similarly, to study
wage level in cotton mill industry of India, separate averages should be calculated
for the male and female workers. Again, adult workers should be separately
studied from the juvenile group. Thus we see that as far as possible, the data from
which an average is calculated should be a homogeneous lot. Homogeneity can be
achieved either by selecting only like items or by dividing the heterogeneous data
into a number of homogeneous groups.
(b) What are the uses of averages?

ANS: The use or application of a particular average depends upon the purpose of
the investigation. Some of the cases of different averages are as follows:

Arithmetic Mean

Arithmetic mean is considered an deal average. It is frequently used in all the


aspects of life. It possesses many mathematical properties and due to this it is of
immense utility in further statistical analysis. In economic analysis arithmetic mean
is used extensively to calculate average production, average wage, average cost,
per capital income exports, imports, consumption, prices, etc. When different items
of a series have different relative importance, then weighted arithmetic mean is
used.

Geometric Mean

Use of Geometric mean is important in a series having items of wide dispersion. It


is used in the construction of index number. The averages of proportions,
percentages and compound rates are computed by geometric mean. The growth of
population is measured in it as population increases in geometric progression.

Harmonic Mean

Harmonic mean is applied in the problems where small items must get more
relative importance than the large ones. It is useful in cases where time, speed,
values given in quantities, rate and prices are involved. But in practice, it has little
applicability.

Median and partition Values


Median and partition values are positional measures of central tendency. There are
mainly used in the qualitative cases like honestly, intelligence, ability, etc. In the
distributions which are positively skewed, median is a more suitable average.
These are also suitable for the problems of distribution of income, wealth,
investment, etc.

Mode

Mode is also positional average. Its applicability of daily problems is increasing.


Mode is used to calculate the 'modal size of a collar', 'modal size of shore', or
'modal size of ready-made garments' etc. It is also used in the sciences of Biology,
Meteorology, Business and Industry.

Q 3. What is test statistic? Why do we have to know the distribution of a test


statistic?
ANS: A statistic is calculated from the sample. To begin with we assume that the
hypothesis about the population parameter is true. We compare the value of the
statistic with the hypothetical value of the parameter. If the difference between
them is small, the hypothesis is accepted and if the difference between them is
large, the hypothesis is rejected. A statistic on which the decision can be based
whether to accept or reject a hypothesis is called test statistic. It is important to
remember that a test-statistic does not prove the hypothesis to be correct but if
furnishes as evidence against the hypothesis. Some of the test statistics to be
discussed later are Z, t and Chi – Square.

Statistics is a diverse subject and thus the mathematics that are required depend on
the kind of statistics we are studying. A strong background in linear algebra is
needed for most multivariate statistics, but is not necessary for introductory
statistics. A background in Calculus is useful no matter what branch of statistics is
being studied, but is not required for most introductory statistics classes.At a bare
minimum the student should have a grasp of basic concepts taught in Algebra and
be comfortable with "moving things around" and solving for an unknown.

Refresher Course

Most of the statistics here will derive from a few basic things that the reader should
become acquainted with.

Absolute Value

If the number is zero or positive, then the absolute value of the number is simply
the same number. If the number is negative, then take away the negative sign to get
the absolute value.

Examples

 |42| = 42
 |-5| = 5
 |2.21| = 2.21

Factorials

A factorial is a calculation that gets used a lot in probability. It is defined only for
integers greater-than-or-equal-to zero as:
Examples

In short, this means that:

0! = 1 =1

1! = 1 · 1 =1

2! = 2 · 1 =2

3! = 3 · 2 · 1 =6

4! = 4 · 3 · 2 · 1 = 24

5! = 5 · 4 · 3 · 2 · 1 = 120

6! = 6 · 5 · 4 · 3 · 2 · 1 = 720

Summation

The summation (also known as a series) is used more than almost any other
technique in statistics. It is a method of representing addition over lots of values
without putting + after +. We represent summation using a big uppercase sigma: ∑.

Examples

Very often in statistics we will sum a list of related variables:


Here we are adding all the x variables (which will hopefully all have values by the
time we calculate this). The expression below the ∑ (i=0, in this case) represents
the index variable and what its starting value is (i with a starting value of 0) while
the number above the ∑ represents the number that the variable will increment to
(stepping by 1, so i = 0, 1, 2, 3, and then 4). Another example:

Notice that we would get the same value by moving the 2 outside of the summation
(perform the summation and then multiply by 2, rather than multiplying each
component of the summation by

2). Infinite series

There is no reason, of course, that a series has to count on any determined, or even
finite value—it can keep going without end. These series are called "infinite series"
and sometimes they can even converge to a finite value, eventually becoming equal
to that value as the number of items in your series approaches infinity (∞).

Examples
This example is the famous geometric series. Note both that the series goes to ∞
(infinity, that means it does not stop) and that it is only valid for certain values of
the variable r. This means that if r is between the values of -1 and 1 (-1 < r < 1)
then the summation will get closer to (i.e., converge on) 1 / 1-r the further you take
the series out.

Linear Approximation
Student-t Distribution at various critical values with varying degrees of freedom.

v / α 0.20 0.10 0.05 0.025 0.01 0.005

40 0.85070 1.30308 1.68385 2.02108 2.42326 2.70446

50 0.84887 1.29871 1.67591 2.00856 2.40327 2.67779

60 0.84765 1.29582 1.67065 2.00030 2.39012 2.66028

70 0.84679 1.29376 1.66691 1.99444 2.38081 2.64790

80 0.84614 1.29222 1.66412 1.99006 2.37387 2.63869

90 0.84563 1.29103 1.66196 1.98667 2.36850 2.63157

100 0.84523 1.29007 1.66023 1.98397 2.36422 2.62589

Let us say that you are looking at a table of values, such as the one above. You
want to approximate (get a good estimate of) the values at 63, but you do not have
those values on your table. A good solution here is use a linear approximation to
get a value which is probably close to the one that you really want, without having
to go through all of the trouble of calculating the extra step in the table.
This is just the equation for a line applied to the table of data. xi represents the data
point you want to know about, is the known data point beneath the one you
want to know about, and is the known data point above the one you want to
know about.

uppose you are sampling from a population with mean and


standard deviation he sample size is n hat are the expected
value and the variance of a sample mean ?

ANS: If sample mean is xbar, E(xbar)=μ=1065

Var(xbar) = (population variance)/n==100^2/100=100

OR

 E[Xbar] = μ = 1,065

 Var(Xbar) = σ^2/n = 500^2/100 = 2500


Assignment Set- 1
Master of Business Administration-MBA Semester

MB0041-Financial and Management Accounting

Q.1 Assure you have just started a Mobile store. You sell mobile sets and
currencies of Airtel, Vodaphone, Reliance and BSNL. Take five transactions
and prepare a position statement after every transaction. Did you firm earn
profit or incurred loss at the end? Make a small comment on your financial
position at the end.

ANS: We shall consider five transactions and show how they are accounted for in the
books of the business.

1. Mr. Rajesh brings Rs.100000 cash as capital into his business.


2. He purchases Mobile Set to his shop Rs.10000
3. He buys currencies for cash Rs.50000
4. He sells currencies worth Rs.30000 for Rs.40000 on credit to Arjun
5. He pays wages to servants Rs.1000

Transaction 1: The business receives capital in cash. Capital is a liability and cash is
an asset to the business.
Liability Asset
Capital 100000 Cash 100000

Transaction 2: Mobile Set is purchased for cash. This transaction can be reflected as
Under.
Q.2a. List the accounting standards issued by ICAI?
ANS: To bring uniformity in terminology, accounting concepts, conventions, and
assumptions, the Institute of Chartered Accountants of India (ICAI) established
Accounting Standards Board (ASB) in 1977. An Accounting Standard is a selected set
of accounting policies or broad guidelines.

Example: While depreciating an asset the practice of adopting straight line method or
diminishing balance method or any other method is a convention regarding the
principles and methods to be chosen out of several alternatives. There are altogether 32
accounting standards issued by ASB out of which, one standard (AS8) has been
withdrawn pursuant to AS26 becoming mandatory.

2b. Write short notes of IFRS?


ANS: IFRS are standards, interpretations and framework for the preparation and
presentation of financial statements. IFRS was framed by International Accounting
Standards Board (IASB).
The objective of financial statement is to provide information about the financial position,
performance and changes in the financial position of an entity. It should also provide the
current financial status of the entity to all the users of financial information. IFRS follows
accrual basis of accounting and the financial statements are prepared on the basis that
an entity will continue for the foreseeable future. IFRS helps entities access global
capital market with ease.

Q.3 Choose an Indian Company of your choice that has adopted Balance Score
Card and detail on it?
ANS: The Balanced Score Card is a framework for integrating measures derived from
strategy. While retaining financial measures of past performance, the Balanced Score
Card introduces the drivers of future financial performance. (Figure 1) The drivers
(customer, internal business process, learning & growth perspectives) are derived from
the organization's strategy translated into objectives and measures.
The Balanced Score Card is more than a measurement system it can be used as an
organizing framework for their management processes. The real power of the Balanced
Score Card is when it is transformed from a measurement system to a management
system. It fills the void that exists in most management systems - the lack of a
systematic process to implement and obtain feedback about strategy

Q.4 What is a cash budget? How it is useful in managerial decision making?


ANS: A proper control over cash is very essential. Cash is an important component in
any activity. The control becomes inescapable. If cash is not properly managed or if it is
mismanaged, the ultimate result would be disastrous. In many times and in many
business situations, business failures are noticed due to the lacunae found in the cash
management. Hence cash budgeting occupies a pivotal place in the study of Financial
Management.
Cash budgeting is the process of forecasting the expected receipts known as cash
inflows, and expected payments known as cash outflows to meet the future obligations.
The written statement of receipts and payments is known as the cash budget. It is a
crystal ball which enables one to observe the future movements in cash position. It is a
mere forecast of cash position of an undertaking for a definite period of time. The period
may be daily, weekly, monthly, quarterly, semi-annually, or annually. The major two
components of cash budget would be forecast first the cash receipts and then second
forecasting the cash disbursements.

The receipts of cash are formatted as follows:


1. Opening balance of cash in hand and cash at bank
The Management Accounting Perspective of the Business Enterprise
The management accounting view of business may be divided into two broad
categories: (1) basic features and (2) basic assumptions.

Basic Features
The business firm or enterprise is an organizational structure in which the basic
activities are departmentalized as line and staff. There are three primary line functions:
marketing, production, and finance. The organization is run or controlled by individuals
collectively called management. The staff or advisory functions include accounting,
personnel, and purchasing and receiving. The organization has a communication or
reporting system (e.g. budgeting) to coordinate the interaction of the various staff
and line departmental functions. The environment in which the organization operates
includes investors, suppliers, governments (state and federal), bankers, accountants,
lawyers, competitors, etc.)

The organizational aspect of the business firm is illustrated in Figure 2.1. This
descriptive model shows that there are different levels of management. A commonly
used approach is to classify management into three levels: Top management, middle
management, and lower level management. The significance of a hierarchy of
management is that decision‑making occurs at three levels.
Basic Assumptions in Management Accounting
The framework of management accounting is based on a number of implied
assumptions. Although no single work has attempted to identify all of the assumptions,
. Five categories of assumptions will be
presented:
1. Basic goals
2. Role of management
3. Nature of Decision‑making
4. Role of the accounting department
5. Nature of accounting information
Basic Goal Assumptions - The basic goals or objectives the business enterprise
may be multiple. For example, the goal may be to maximize net income. Other goals
could be to maximize sales, ROI, or earnings per share. Management accounting
does not require a specific of type of goal. However, whatever form the goal takes,
management will at all times try to achieve a satisfactory level of profit. A less than
satisfactory level of profit may portend a change in management.
Role of Management Assumptions - The success of the business depends
primarily upon the skill and abilities of management–which skills can vary widely
among different managers. The business is not completely at the mercy of market
forces. Management can through its actions (decisions) influence and control events
within limits. In order to achieve desired results, management makes use of specific
planning and control concepts and techniques. Planning and control techniques
which management may use include business budgeting, cost‑volume‑profit
analysis, incremental analysis, flexible budgeting, segmental contribution reporting,
inventory models, and capital budgeting models. Management, in order to improve
decision‑making and operating results, will evaluate performance through the use of
flexible budgets and variance analysis.
Decision‑making Assumptions ‑ A critical managerial function is decisionmaking.
Decisions which management must make may be classified as marketing,
production, and financial. Decisions may also be classified as strategic and tactical
and long‑run and short‑run. A primary objective of decision‑making is to achieve
optimum utilization of the business’s capital or resources. Effective decision‑making
requires relevant information and special analysis of data.
Accounting Department Assumptions ‑ The accounting department is a primary
source of information necessary in making‑decisions. The accounting department
is expected to provide information to all levels of management. Management will
consider the accounting department capable of providing data useful in making
marketing, production, and financial decisions.
Nature of Accounting Information - In order for the accounting department to
make meaningful analysis of data, it is necessary to distinguish between fixed and
variable costs and other types of costs that are not important in the recording of
business transactions. Some but not all of the information needed by management can
be provided from financial statements and historical accounting records. In addition to
historical data, management will expect the management accountant to provide other
types of data, such as estimates, forecasts, future data, and standards. Each specific
18 | CHAPTER TWO • Management Accounting and Decision-Making
managerial technique requires an identifiable type of information. The accounting
department will be expected to provide the information required by a specific tool. In
order for the accounting department to make many types of analysis, a separation of
costs into fixed and variable will be required. The management accountant need not
provide information beyond the relevant range of activity.

Q.5 State the importance of differentiating between the fixed costs and variable
costs in managerial decision?

ANS: Fixed Costs: These are those costs which remain fixed up to certain range of work
capacity no matter how much product you produce within that capacity range. Like factory
building rent. You pay the rent no matter that did you use that building for making the products
or not.

Variable Costs: These are those costs which change with the change in the number of product
units you produce. Like Material , Labor etc

Mixed Cost/Semi Variable Costs: These are those cost the part of which is remain fixed and
some part of the cost is variable.

Fixed vs. variable costs


Understanding the process of managing costs first requires an understanding of two general types of
costs: fixed and variable.

Fixed costs, or sunk costs as they are sometimes called, are those that generally do not vary between
payment intervals. Generally, these costs cannot be altered on a short-term basis because of
contractual agreements or simply because it is impractical.

Table 1: Examples of fixed and variable costs

Fixed monthly costs

Rent.............................................................................. $1,100

Yellow pages ad............................................................ $400

Loan payment................................................................ $500

Office manager’s salary.................................................. $3,000

Doctor’s salary.............................................................. $5,000

Total fixed costs............................................................ $10,000

Variable costs per patient visit

Supplies (forms, covers, etc.)......................................... $3

Electricity to operate a roller table.................................. $1

Collections costs (stamps, invoices, etc.)........................ $1

ne way to determine your fixed costs is to consider the expenses you would continue to incur if you
temporarily closed your practice and no patients were being treated. In this case, your rent, car
leases, yellow page ad fees and loan payments would still be due. They generally do not change with
increases or decreases in business activity.

It is important to note that fixed costs are unvarying only within a certain range of business activity.
For example: if the practice grows enough to require additional space or additional employees, the
fixed costs associated with rent or salaries will change as well.

Variable costs are those that change as the level of business activity changes. Examples of the
variable costs within a chiropractic business would be supplies used for each patient visit, collection
fees paid to external billing agencies and wages for hourly, part-time employees. These costs are
driven primarily by the practice’s business activity, by number of patients that the practice treats.
The “closed practice” test we used above to determine the fixed costs of a practice can also be used to
determine the variable costs. The variable costs are those that would stop if the practice were closed
for a month and no patients were treated at all.

Once you understand the difference between fixed and variable costs, it is important to know how to
distinguish one from the other. For instance, consider a practice that has fixed costs of $10,000 and
variable costs of $5 per patient. (See Table 1 for an example of fixed and variable costs.)

To cover its monthly expenses, the practice would have to earn $10,000 in fees plus $5 per patient
treated. If the practice had only one patient visit per month, it would have to charge $10,005 for that
one treatment in order to cover its fixed and variable costs!

If the practice had 1,000 patient visits during the month, its total costs would be $15,000 ($10,000 in
fixed costs plus 1,000 patient visits at $5 each). Therefore, this practice would only have to charge
$15 per patient visit to cover its fixed and variable costs.

Q.6 Identify the users of accounting information?

ANS: There are two types of users( internal and external):- list of internal
users:-

1. employees
2. management
3. shareholders/owners.

list of external users :-

those who have economic transactions like

 suppliers
 creditors
 bankers
 financial institutions
others like

 competitors
 government and regulatory agencies
 auditors
 researchers and academicians
 representatives of others interest like brokers ,underwriters etc
 potential shareholders

* Management - obtain financial information in a way that best suits its needs, (i.e. ways to cut
costs, maximize profit, etc.).
* Board of Directors - information to assist in determination of current executive positions

External users:
* IRS - requires the information be presented in a very specific manner.
* Stockholders and potentional stockholders - information that will aid in decision continue to
hold the stock, sell the stock, or purchase the stock.
* Bondholders, bankers & other creditors - information that will aid in decision of whether or
not to purchase the bond. Bank needs information that will help it determine the company's
ability to repay the loan and interest.
* Employee (can also be considered internal) - information that will aid in decision to continue
working at the company or look for employment elsewhere.
* Supplier - information to aid in decision to continue or start supplying to the company.
Master of Business Administration - MBA Semester I

MB0042 – Managerial Economics

Q.1 Income elasticity of demand has various applications. Explain each


application with the help of an example.

Ans :Income elasticity of demand may be defined as the ratio or proportionate


change in the quantity demanded of a commodity to a given proportion change in
the income. In short, it indicates the extent to which demand changes with a
variation in consumer‘s income. Thefollowing formula helps to measure the
income elasticity (Ey).

Or

Where

· Ey is income elasticity of demand

· D is change in demand

· D is original demand

· Y is change in income

· Y is original income

Example

Original demand=400 units Original income= 4000 units

New demand =700 units New income= 6000 units

Change in demand= 700-400= 300 units change in income=6000-4000=2000


Hence Ey=300/2000*4000/400=1.5

Generally speaking Ey is positive. This is because there is a direct relationship


between income and demand, i.e. higher the income; higher would be the demand
and vice versa. On the basis of the numerical value of the co-efficient, Ey is
classified as greater than one, less than one, equal to one, equal to zero and
negative. The concept of ey helps us in classifying commodities in to

different categories.

1. When Ey is positive, the commodity is normal (used in day-to-day life)

2. When Ey is negative, the commodity is inferior. ( for example jowar, beedi etc)

3. When Ey is positive and greater than one, the commodity is luxury.

4. When Ey is positive but less than one, the commodity is essential.

5. When Ey is zero, the commodity is neutral. E.g. salt, match box etc.

Practical application of income elasticity of demand

1. Helps in determining the rate of growth of the firm.

If the growth rate of the economy and income growth of the people is reasonable

forecasted, in that case it is possible predict expected increase in the sales of a firm
and vice versa.
2. Helps in the demand forecasting of a firm.

It can be in estimating future demand provided the rate of increase in income and
Ey for the products are known. Thus, it helps in demand forecasting activities of a
firm.

3. Helps in production planning and marketing.

The knowledge of Ey is essential for production planning, formulating marketing

strategy, deciding advertising expenditures and nature of distribution channel etc in


the long run.

4. Helps in ensuring stability in production.

Proper estimation of different degrees of income elasticity of demand for different


types of product helps in avoiding over-production or under-production of a firm.
One should know whether rise or fall in income is permanent or temporary.

5. Helps in estimating construction of houses.

The rate of growth in incomes of people also helps in housing programs in a


country.

Thus it helps a lot in managerial decisions of a firm.

Q.2 When is the opinion survey method used and what is the effectiveness of
the method.

Ans :Survey of buyer‘s intention or preference is one of the important methods of


demand forecasting. It is also called ―Opinion Survey Method‖.

Under this method, consumer buyers are requested to indicate their preference and
willingness about a particular product. They are about to reveal their future
purchase plans with respect to specific items.

They are expected to give answer to question like what items they intends to buy,
in what quantity, why, where, what quality they expect, how much they are
planning to spend etc.

Generally, the field surveys are conducted by the marketing research departments
of the company or hiring the services of outside research organization consisting of
learned and highly qualified professionals.

The heart of the survey is questionnaire. It is a comprehensive one covering almost


all questions either directly or indirectly in a most intelligent manner. It is prepared
by an expert body who are specialist in the field or marketing.

The questionnaire is distributed among the consumer either through mail or in


person by the company. Consumers are requested to furnish all relevant and correct
information.

The next step is to collect the questionnaire from the consumers for the purpose of

evaluation. The materials collected will be classified, edited and analyzed. If any
bias prejudices, exaggerations, artificial or excess demand creation are found at the
time of answering they would be eliminated.

The information so collected will now be consolidated and reviewed by the top

executives with lot of experiences. It will be examined thoroughly. Inferences are


drawn and conclusions are arrived at. Finally a report is prepared and submitted to
the management for taking final decisions.

The success of the survey method depends on many factors:


1. The nature of the question asked.

2. The ability of the surveyed.

3. The representative of the sample

4. Nature of the product

5. Characteristics of the market

6. Consumer behavior

7. Techniques of analysis

8. Conclusion drawn etc.

The management should not entirely depend on the result of survey reports t
project future demand. Consumer may not express their honest and real views and
as such they may give only the broad trends in the market. In order to arrive, at
right conclusion, field surveys should be regularly checked and supervised.

This method is simple and useful to the producers who produce goods in bulk.
Here the burden of forecasting is put on the customers.

However this method is not much useful in estimating the future demand of the

household as they run in a large numbers and also do not freely express their future
demand requirements. It is expensive and so difficult. Preparation of questionnaire
is not an easy task.

At best it can be used for short term forecasting.


Q.3 Show how price is determined by the forces of demand and supply, by
using forces of equilibrium.

Ans The word equilibrium is derived from the Latin word ―aequilibrium‖ which

means equal balance. It means a state of even balance in which opposing forces or
tendencies neutralize each other. It is a position of rest characterized by absence of
change. It is a state where there is complete agreement of the economic plans of
the various market participants so that no one has a tendency to revise or alter his
decision. In the words of professor Mehta: ―Equilibrium denotes in economics
absence of change in movement.‖

Market Equilibrium

There are two approaches to market equilibrium viz., partial equilibrium approach

and the general equilibrium approach. The partial equilibrium approach to pricing

explains price determination of a single commodity keeping the prices of other

commodities constant. On the other hand, the general equilibrium approach


explains the mutual and simultaneous determination of the prices of all goods and
factors. Thus it explains a multi market equilibrium position.

Earlier to Marshall, there was a dispute among economists on whether the force of

demand or the force of supply is more important in determining price. Marshall


gave equal importance to both demand and supply in the determination of value or
price. He compared supply and demand to a pair of scissors – ― We might as
reasonably dispute whether it is the upper or the under blade of a pair of scissors
that cuts a piece of paper, as whether value is governed by utility or cost of
production. ‖ Thus neither the upper blade nor the lower blade taken separately can
cut the paper; both have their importance in the process of cutting. Likewise
neither supply alone, nor demand alone can determine the price of a commodity,
both are equally important in the determination of price. But the relative
importance of the two may vary depending upon the time under consideration.
Thus, the demand of all consumers and the supply of all firms together determine
the price of a commodity in the market.

Equilibrium between demand and supply price:

Equilibrium between demand and supply price is obtained by the interaction of


these two forces. Price is an independent variable. Demand and supply are
dependent variables. They depend on price. Demand varies inversely with price, a
rise in price causes a fall in demand and a fall in price causes a rise in demand.
Thus the demand curve will have a downward slope indicating the expansion of
demand with a fall in price and contraction of demand with a rise in price. On the
other hand supply varies directly with the changes in price, a rise in price causes a
rise in supply and a fall in price causes a fall in supply. Thus the supply curve will
have an upward slope.At a point where these two curves intersect with each other
the equilibrium price is established. At this price quantity demanded is equal to the
quantity demanded. This we can explain with the help of a table and a diagram

Price in rs Demand in Supply in units State of Pressure on


units market price
30 5 25 D<S P decreases
25 10 20 D<S P decreases
20 15 15 D=S Neutral
10 20 10 D>S P increases
5 30 5 D>S P increases
.

In the table at Rs.20 the quantity demanded is equal to the quantity supplied. Since
the price is agreeable to both the buyer and sellers, there will be no tendency for it
to change; this is called equilibrium price. Suppose the price falls to Rs.5 the buyer
will demand 30 units while the seller will supply only 5 units. Excess of demand
over supply pushes the price upward until it reaches the equilibrium position
supply is equal to the demand. On the other hand if the price rises to Rs.30 the
buyer will demand only 5 units while the sellers are ready to supply 25 units.

Sellers compete with each other to sell more units of the commodity. Excess of
supply over demand pushes the price downward until it reaches the equilibrium.
This process will continue till the equilibrium price of Rs.20 is reached. Thus the
interactions of demand and supply forces acting upon each other restore the
equilibrium position in the market.

In the diagram DD is the demand curve, SS is the supply curve. Demand and
supply are in equilibrium at point E where the two curves intersect each other. OQ
is the equilibrium output. OP is the equilibrium price. Suppose the price OP2 is
higher than the equilibrium price OP. at this point price quantity demanded is
P2D2. Thus D2S2 is the excess supply which the seller wants to push into the
market, competition among the sellers will bring down the price to the equilibrium
level where the supply is equal to the demand. At price OP1, the buyers will

demand P1D1 quantity while the sellers are ready to sell P1S1. Demand exceeds
supply. Excess demand for goods pushes up the price; this process will go until
equilibrium is reached where supply becomes equal to demand.

Q.4 Distinguish between fixed cost and variable cost using an example.

Ans: Fixed cost: These costs are incurred on fixed factors like land, building,
equipments, plants, superior types of labour, top management etc. fixed costs in the
short run remains constant because the firm does not change the size of plant and
the amount of the fixed factors employed. Fixed costs do not vary with either
expansion or contraction in output. These cost are to be incurred by a firm even
output is zero. Even if the firm close down its operation for some

time temporarily in the short run, but remains in business, these cost have to be
borne by it.

Hence, these costs are independent of output and are referred to as unavoidable
contractual cost.

Prof. Marshall called fixed cost as supplementary costs. They include such items as
contractual rent payments, interest on capital borrowed, insurance premium,
depreciation and maintenance allowance, administrative expenses like manager‘s
salary or salary of the permanent staff, property and business taxes, license fees,
etc. They are called as over- head costs because these costs are to incurred whether
there is production or not. These costs are to be distributed on each units of output
produced by a firm. Hence, they are called as indirect costs.

Variable Costs: The costs corresponding to variable factors are described as


variable costs. These costs are incurred on raw materials, ordinary labour,
transport, power, fuel, water etc, which directly vary in the short runs.

Variable costs are directly and proportionately increases or decreases with the level
of output. If a firm shut down for some times in the short run; then it will not use
the variable factors of production and will not therefore incurs any variable costs.
Variable costs are incurred only when some amount of output is produced. Total
variable cost increases with the level of increase in the level of production and
vice-versa. Prof. Marshall called variable costs as prime costs or direct costs
because the volume of output produced by a firm depends directly upon them.

It is clear from the above description that a production cost consists of both fixed
as well as variable costs. The difference between the two is meaningful and
relevant only in the short run. In the long run all costs become variable because all
factors of production become adjustable and variable in the long run.

However, the distinction between the fixed and variable costs is very important in
theshort because it influences the average costs behavior of the firm. In the short
run, even if a firm wants to close down its operation but wants to remain in the
business, it will have to incur fixed costs but it must cover at least its variable
costs.
Q.5 Discuss Marris Growth Maximization model ?

Profit maximization is traditional objective of a firm. Sales maximization objective


is explained by Prof. Boumal. On similar lines, Prof. Marris has developed another
alternative growth maximization model in recent years. It is a common factor to
observe that each firm aims at maximizing its growth rate as this goal would
answer many of the objectives of a firm.

Marris points out that a firm has to maximize its balanced growth rate over a
period of time.

Marris assumes that the ownership and control of the firm is in the hands of two
groups of people, i.e. owner and managers. He further points out that both of them
have two distinctive goals. Managers have a utility function in which the amount
of salary, status, position, power, prestige and security of job etc are the most
import variable where as in case of are more concerned about the size of output,
volume of profits, market shares and sales maximization.

Utility function of the manager and that the owner are expressed in the following
manner-

Uo= f [size of output, market share, volume of profit, capital, public esteem etc.]

Um= f [salaries, power, status, prestige, job security etc.]

In view of Marris the realization of these two functions would depend on the size
of the firm.

Larger the firm, greater would be the realization of these functions and vice-versa.
Size of the firm according to Marris depends on the amount of corporate capital
which includes total volume of the asset, inventory level, cash reserve etc. He
further points out that the managers always aim at maximizing the rate of growth
of the firm rather than growth in absolute size of

the firms. Generally managers like to stay in a grouping firm. Higher growth rate
of the firm satisfy the promotional opportunity of managers and also the share
holders as they get more dividends.

Q.6 Explain how fiscal policy is used to achieve economic stability.

Ans:
In order to achieve a stable economic condition, fiscal policy has to play a positive
and constructive role both in developed and developing nations. The specific role
to be played by fiscal policy can be discussed as follows:
To act as optimum allocator of resources: As most of the resources are
scarce in their supply, careful planning is needed in its allocation so as to achieve
the set targets.
Rational allocation would ensure fulfillment of various objectives.
To act as a saver:
1. It should follow a rational consumption policy reduces the MPC and raises the
MPS.
2. Taxation policy has to be modified to raise the rates of old taxes, introduces new
additional taxes, and extends the tax-nets.

3. Profit earning capacity of public sector units are to be raise substantially to mop-
up financial resources.

4. The government should borrow more money both in the country and outside the
country.
5. Higher the rate of interest are to be offered for government bonds and security.

· To act as an investor: Mere mobilization of financial resources is not an end in


itself. It should result in the creation of real resources which are more important in
accelerating the growth process. Rapid economic growth depends upon the volume
of investment.

Hence, fiscal policies have to be ensuring higher volume of investment in both


private and public sectors.

· To act as price stabilizer: price stability is of paramount of importance in an


economy.Extreme levels of both inflation and deflation would disrupt and disturb
the normal and regular working of an economic system. This would come in the
way of stable and persistent growth. Hence all measures are to be taken to check
these two dangerous situations so as to create necessary congenial atmosphere to
prepare the background for rapid economic growth.

· To act as an economic stabilizer: Price stability would create the necessary


background for over all economics stability. Upswing and downswing in the level
of economic activities are to be avoided. If an economy is subject to frequent
fluctuation in the form of trade cycle, certainly, it would undermine and disturb the
growth process. Instability would come in the way of persistent and consistent
growth in a country. Hence all measure to be taken to ensure economic stability.

· To act as an employment generator: Fiscal policy should help in mobilizing


more financial resources, convert them in to investment and create more
employment opportunity to absorb the huge unemployed man power.

· To act as balancer: There must be proper balance between aggregate saving and
aggregate investment, demand and supply, income and output and expenditure,
economic overhead capital and social overhead capital etc. Any sort of imbalance
would result in either surpluses or scarcity in different sectors of the economy
leading to fast growth in some sectors followed by lagging of some other sectors.

To act as growth promoter: The basic objective of any economic policy is to


ensure higher economic growth rates. This is possible when there is higher national
savings, investment, production, employment and income. Hence, fiscal policy is
to be designed in such a manner so as to promote higher growth in an economy.

· To act as in come redistribute: Fiscal policy has to minimize inequalities and


ensure distributive justice in an economy. This is possible when a rational taxation
and public expenditure policy is adopted. More money is collected from richer
section of the society through various imaginative taxation policies and a larger
amount of money is to be spent in favor of poorer sections of the society. Thus,
inequality is to be reduced to the minimum.

· To act as stimulator of living standards of people: the final objective is to raise


the level of living standards of the people. This is possible when there is higher
output, income and employment leading to higher purchasing power in the hands
of common man. Hence, fiscal policy should help in creating more wealth in the
economy. If there is economic prosperity, then it is possible to have a satisfactory,
contended and peaceful life.

Thus, fiscal policy has to play a major role in promoting economic growth in a
country.
Master of Business Administration-MBA Semester I
Subject Code – MB0043
Subject Name – Human Resource Management

Q.1 Write down the difference between Personnel management and Human
Resource management.[10 Marks]

Ans

• Personnel management is workforce centred, directed mainly at the


organization‘s employees; such as finding and training them, arranging for them to
be paid, explaining management‘s expectations, justifying management‘s actions
etc. While on the other hand, HRM is resource –centered, directed mainly at
management, in terms of devolving the responsibility of HRM to line management,
management development etc.

• Although indisputably a management function, personnel management has never


totally identified with management interests, as it becomes ineffective when not
able to understand and articulate the aspirations and views of the workforce, just as
sales representatives have to understand and articulate the aspirations of the
customers.

• Personnel Management is basically an operational function, concerned primarily


with carrying out the day-to day people management activities. While on the other
hand, HRM is strategic in nature, that is, being concerned with directly assisting an
organization to gain sustained competitive advantage.
HRM is more proactive than Personnel Management. Whereas personnel
management is about the maintenance of personnel and administrative systems,
HRM is about the forecasting of organizational needs, the continual monitoring
and adjustment of personnel systems to meet current and future requirements, and
the management of

change.

Q.2 Write a note on scope of HR in India. [10 Marks]

Ans Its only in the past 10-12 years with the immense growth on account of the
IT industry that winds of change began to blow. It was largely the advent of the
Information Technology era in India that brought with it the western management
practices. MNC‘s (multinational companies) started up their operations in India.
The FDI (foreign direct investment) went up steeply as the world saw the potential
in the country‘s human resources. India became a preferred location for MNC‘s
primarily from the USA, followed by other developed countries.
The gave birth to a new generation of management as well as HRM practices. New
hiring methods, new ways of paying salaries, new employment terms and most
importantly increased focus on individual performance and outcomes. There was
emphasis on deliverables and linking individual and team performance to business
results and success. Given the highly educated workforce there was a de-emphasis
in the role of the trade unions. The era of the trade union dominance gave way to
the new order of individual negotiated salaries and terms and clearly performance
linked assessment systems. Another transformation that the Indian workplace
witnessed was the focus on ethics and ethical practices in doing business. It was
only fair to expect that with the weak legal system, it needed the support of the
government policies and the corporate policies to beat the corruption that existed.
This has significantly contributed to India emerging as a preferred destination for
doing business.
All of this has yielded to give way to the birth of the ‗professional manager‘
Professional managers today are a critical and essential part of the Indian
corporate. The professional manager brought about a shift in the culture from a
highly authoritarian approach of getting work done, to a more collaborative and
participative approach. The entrepreneurs who earlier operated in a secure,
sheltered market and hardly face challenges, were challenged by the globalization
that swept in with the liberalization policies and measure brought in by the Indian
government late 1995 and onwards. Despite the challenges, the Indian employee
and his manager evolved. Together they stepped up to face the challenge head-on
and to win not only in India but also globally. The levers of (a) low cost, (b) highly
skilled, and (c) English as the medium of education and it being the corporate-
language: were the key drivers that enabled the flow of global business to India.
There was exponential growth in employment both directly (jobs in the
international and domestic companies) as well as indirectly (as support industries
like transport, catering and ancillary industries). The simultaneous investment of
the government in building the necessary infrastructure did its share of providing
impetus towards creating more jobs for the people of the country.
Hence, human relations movement in India has evolved very differently as
compared to what we see in the developed economies of the USA and the UK.
What is currently acting as a limitation is the enhanced awareness on the need for
research based HRM practices. While there is a lot of work happening in the Indian
education system to promote this, it is going to take a while before it can create a
distinct body of knowledge that is referenceable. For now the industry relies on
emulating westerns HR practices and customizing on a as-needed basis for the
Indian corporation. For the rest the industry forums and consortiums like the
NASSCOM act as a hub bringing together organizations on a regular basis to
discuss challenges and share best practices and identify ways and means o
overcome them together. So far this has been successful and working to the
advantage of the Indian corporate. Leading MNC research and consulting firms
like Mercer and Hewitt too contribute to the industry through carrying out research
and sharing reports on a regular basis. The approach however remains analytical
and less prescriptive.

Q.3 Explain the critical steps in Human Resource Planning system .[10
Marks]

Ans Human Resource planning can be defined as a process by which an


organization ensures thatit has the right number and kinds of people, at the right
place, at the right time, capable ofeffectively and efficiently completing those tasks
that will help the organization achieve itsoverall objectives or in other words HRP
can be defined as planning for the future personnelneeds of an organization, taking
into account both internal activities and factors in theexternal environment.

Need and Importance of HRP


Human resource Planning translates the organization objectives and plans into the
number of workers needed to meet these objectives. The need and importance of
HRP is as follows:
determining the future manpower requirements and avoids
problems like over staffing or understaffing in the organization.

policies etc. that generates changes in the job content, skill requirements and
number and types of personnel required.
staff becomes redundant, the HR manager has to attract and retain qualified and
skilledpersonnel and also required to deal with issues like career development,
successionplanning for which he takes the help of HRP.

employment and promotional appointments to the candidates fro weaker sections,


physically handicapped and socially and politically oppressed citizens.

execution of personnel functions like recruitment, selection, transfers, promotions,


layoffs, training and development and performance appraisal.

turn will facilitate reduction in personal costs.

assignments to develop managers and to ensure the organization has a steady


supply of experienced and skilled employees.

Factors affecting Human Resource Planning

HRP is a dynamic and on going process. The process of updating is not very
simple, since HRP is influenced by many factors, which are as follows:

of staff needed.

adopted by it. For e.g. the growth of a business calls for hiring of additional labour,
while mergers will need a plan for layoffs.
carefully formulate the HR policies and so the HR manager has to evolve
suitablemechanism to deal with uncertainties through career developments,
successionplanning, retirement schemes etc.
-term
plans are adopted. And this time span is based on the degree of environmental
uncertainties.

factor influencing HRP. Accurate and timely human resource information system
helps in getting better quality personnel.
ensure that suitable candidates should be appointed at the
right kind of job.
So these are some of the factors that affect the human resource planning.

Limitations of Human Resource Planning


s of an organization,
as future is always uncertain.

resources.

approach and at the


same time are conscious about the changing environment.

turnover
Q.4 List and explain the sources of recruitment? [10 Marks]

Ans : The sources of employee‘s recruitment can be classified into two


types,internal and external. Filing a job opening from within the firm has
advantages of stimulating preparation for possible transfer of promotion,
increasing the general level of morale, and providing moreinformation about job
candidates through analysis of work histories within the organisation.A job posting
has number of advantages. From the view point of employee, it providesflexibility
and greater control over career progress. The jobs posted on notice boards, group
emails and publishing in internal news papers. Internal applications often restricted
to certain employees, the guidelines for companies include:

-Good or better in most recent performance review


-Dependable attendance record
-Not under probationary sanction
-Having been in present position for at least one year

The personnel department acts as a clearing house in screening applications that


areunrealistic, preventing an excess number of bids by a single employee and
counsellingunsuccessful employees in their constant attempt to change their jobs.

Similarly the firm may go to external sources for recruitment of lower entry jobs,
forexpansion, and positions whose specifications cannot be met by the present
personnel. Thefollowing external resources are available for firms:

a)Advertising: There is a trend toward more selective recruitment in advertising.


This canbe affected in at least two ways. First advertisements can be placed in
media read only byparticular groups. Secondly, more information about the
company, the job, and the jobspecification can be included in the ad to permit
some self-screening.
b)Employment Agencies: Additional screening can be affected through the
utilization of employment agencies, both public and private. Today, in contrast to
their formerunsavoury reputation, the public employment agencies in several states
are well-regarded,particularly in the fields of unskilled semi-skilled and skilled
operative jobs. In thetechnical and professional areas, however, the private
agencies tend to specialize in aparticular engineer.

c)Employee Referrals: Friends and relatives of present employees are also a good
source from which employees may be drawn. When the labour market is very tight,
largeemployers frequently offer their employees bonus or prizes for any referrals
that are hiredand stay with the company for a specific length of time. Some
companies maintain aregister of former employees whose record was good to
contact them when there are newjob openings for which they are qualified. This
method of recruitment, however, suffersfrom a serious defect that it encourages
nepotism, i.e.Persons of one‘s community orcaste are employed, who may or may
not be fit for the job.

d)Schools, colleges and professional institutions: These offer opportunities for


recruiting their students. They will also have separate placement cell where the bio
data and otherparticulars of the students are available. The prospective employers
can reviewcredentials and interview candidates for management trainees or
probationers. This is anexcellent source of potential employees for entry-level
positions in the organisations

e)Labour Unions: Firms which closed or union shops must look to the union in
their recruitment efforts. This has disadvantage of monopolistic workforce.

f)Casual applicants: Unsolicited applications, both at the gate and through the
mail, constitute a much-used source of personnel. These can be developed through
attractive employment office facilities, prompt and courteous reply to unsolicited
letters.

g)Professional organisations or recruiting firms or executive


recruiters:Maintain complete records about employed executives. These firms are
looked upon as headhunters, raiders and pirates by organizations may employ
―executive search firms‖ to helpthem find talent. These consulting firms
recommend persons of high calibre formanagerial, marketing and production
engineers‘ posts.

h)Indoctrination seminars for colleges are arranged to discuss the problem of


companies and employees. Professors are invited to take part of these seminars.
Visits to plants arearranged so that professors may be favourably impressed. They
may speak well of acompany and help it in getting the required personnel.

i)Unconsolidated applications: for positions in which large numbers of


candidates are notavailable from other sources, the companies may gain keeping
files of applicationsreceived from candidates who make direct enquiries about the
possible vacancies on theirown, or may send unsolicited applications. This would
be helpful to firms for futurevacancies.

j)Nepotism:the hiring of relatives will be an inevitable component of recruitment


programmes in family-owned firms, such a policy does not necessarily coincide
withhiring on the basis of merit, but interest and loyalty to the enterprise are
offsettingadvantages.

k)Leasing: to adjust to short term fluctuations in personnel needs, the possibility of


leasingpersonnel by the hour or day should be considered. This principle has been
particularlywell developed in the office administration field because they can avoid
any obligation inpensions, insurance and any other fringe benefits.
l)Voluntary Organisations: Such as private clubs, social organisations might also
provideemployees – handicaps, widowed or married women, old persons, retired
hands etc. Inresponse to advertisements.

m)Computer Data Banks: when a company desires a particular type of


employees, jobspecifications and requirements are fed to computers, where they
are matched againstdata stored in. This method is very useful in identifying
candidates for hard-t-fit positionswhich calls for unusual combinations of skills.

Q.5 Describe the grievance handling procedure.

Ans Handling a grievance

When dealing with a grievance:

 ensure you're familiar with the procedure and apply it correctly

 hold any grievance hearing in private without interruptions

 where a grievance relates to the person's line manager, ensure that the
employee can raise the grievance with someone else

 listen carefully to the person's explanation of the problem and consider


whether there is a deeper issue which might be the root cause of the
grievance

 listen to any conflicting points of view

 weigh up all evidence to see whether there is an issue you need to address

 decide what action to take, trying to balance fairness to the person without
compromising the business or other workers

 inform all concerned parties of your decision and the appeal process
 ensure you resolve any problems relating to policies, procedures or conduct
where the grievance procedure highlights these

 keep the process as confidential as possible

Deal with grievances sensitively, particularly where they concern other workers.
You may wish to develop specific procedures for very sensitive matters involving
unfair treatment eg, discrimination, bullying or harassment. . Consider also having
a separate "whistleblowing" procedure, so that workers are encouraged to raise any
complaints about wrongdoing eg fraud, internally rather than disclosing them
outside the business.

If a worker raises a separate grievance during a disciplinary hearing, it's good


practice to adjourn the hearing until the grievance is dealt with.

By dealing with problems in a fair and reasonable manner, you're much less likely
to lose valued and skilled staff through resignation. It will also help you
successfully defend a constructive dismissal claim.

Q 6 what are objectives of training?

Ans

Training objectives are one of the most important parts of training program.
While some people think of training objective as a waste of valuable time. The
counterargument here is that resources are always limited and the training
objectives actually lead the design of training. It provides the clear guidelines
and develops the training program in less time because objectives focus
specifically on needs. It helps in adhering to a plan.
Training objective tell the trainee that what is expected out of him at the end
of the training program. Training objectives are of great significance from a
numberofstakeholderperspectives,

1.Trainer
2.Trainee
3.Designer
4.Evaluator

Trainer – The training objective is also beneficial to trainer because it helps


the trainer to measure the progress of trainees and make the required
adjustments. Also, trainer comes in a position to establish a relationship
between objectives and particular segments of training.
Trainee – The training objective is beneficial to the trainee because it helps in
reducing the anxiety of the trainee up to some extent. Not knowing anything or
going to a place which is unknown creates anxiety that can negatively affect
learning. Therefore, it is important to keep the participants aware of the
happenings, rather than keeping it surprise.

Secondly, it helps in increase in concentration, which is the crucial factor to make


the training successful. The objectives create an image of the training
program in trainee‘s mind that actually helps in gaining attention.

Thirdly, if the goal is set to be challenging and motivating, then the likelihood of
achieving those goals is much higher than the situation in which no goal is set.
Therefore, training objectives helps in increasing the probability that the
participants will be successful in training.

Designer – The training objective is beneficial to the training designer because if


the designer is aware what is to be achieved in the end then he‘ll buy the training
package according to that only. The training designer would then look for the
training methods, training equipments, and training content accordingly to achieve
those objectives. Furthermore, planning always helps in dealing effectively in an
unexpected situation. Consider an example; the objective of one training program
is to deal effectively with customers to increase the sales. Since the objective is
known, the designer will design a training prog that will include ways to
improve the interpersonal skills, such as verbal and non verbal language, dealing
in unexpected situation i.e. when there is a defect in a product or when a customer
angry.
Therefore, without any guidance, the training may not be designed appropriately.

Evaluator – It becomes easy for the training evaluator to measure the progress of
the trainees because the objectives define the expected performance of trainees.
Training objective is an important to tool to judge the performance of participants.

You might also like